[obm-l] [avast! - INFECTED] Delivery by mail

2006-02-06 Por tôpico Villard

Thanks for  use of our  software.





---
avast! Antivirus: Outbound message INFECTED:
\Jol03.zip#4241478289 (Win32:Beagle-IA [Wrm]) was deleted from the message.

Virus Database (VPS): 0606-0, 04/02/2006
Tested on: 6/2/2006 12:37:31
avast! - copyright (c) 1988-2005 ALWIL Software.
http://www.avast.com








Re: [obm-l] Inversa e Transposta + FUNCAO EUREKA

2004-04-27 Por tôpico Villard
Para ficar mais fácil de escrever, seja B = A^(-1). Quero
mostrar que B^t=(A^t)^(-1), ou seja, que B^t * A^t = IMas isso é
verdade, pois B^t * A^t = (A*B)^t = I^t = I , pois B é a inversa de A.
Bem, pessoal, eu andei vendo alguns discutindo o problema 83 da
eureka, aquele das funções : f(2003) = 2003, f(m)=2003 para todo m =
2003, f(m+f(n))=f(f(m)) + f(n).Eu achei 3 funções...Eram f(m) =
m, f(m) = 2003*(1+parte inteira [(m-1)/2003] ),f(m) = 2003*(parte
inteira [m/2003]).Vou ver se escrevo a minha solução num email ainda
hj.Abraços, Villard

- Mensagem Original De:
[EMAIL PROTECTED]Para: "Grupo OBM"
[EMAIL PROTECTED]Assunto: [obm-l] Inversa e
TranspostaData: 27/04/04 16:00

Mais uma de
algera linear...

"Prove que, se A
eh invertivel, entao A(t) eh invertivel e [A(t)] ^ -1 = (A
^-1)(t)"

A(t) =
transposta de A

[]s


Cloves

=
Instruções para entrar na lista, sair da lista e usar a lista em
http://www.mat.puc-rio.br/~nicolau/olimp/obm-l.html
=


[obm-l] Problema da função da eureka 18

2004-04-27 Por tôpico Villard
Como prometido, segue minha solução para o problema 83 da
eureka 18 (colocarei resumida, pois é meio longa):Seja (*)
f(m+f(n))=f(f(m)) + f(n). Faça m=n=0: Isso nos dá f(0)=0.Faça n=0:
f(m)=f(f(m)). Então (*) vira f(m+f(n)) = f(m)+f(n).Seja I =
{0,1,2,...,2002}.Caso 1: f se anula todo o conjunto I.Fazendo
n=2003, temos que f(m+2003) = f(m) + 2003, logo os valores de f são 0,0,
...,0,2003,2003, ...,2003,4006,4006, ... 4006,6009, ou seja, f(m) =
2003*(parte inteira [m/2003])Caso 2: f não se anula em todo o
conjunto I.Seja u o menor valor positivode f(I). Então existe k em
I tal que f(k) = u. Fazendo n = k, temos que f(m+u)=f(m)+u . Usando isso
repetidas vezes, temos que f(j*u) = j*u, para todo inteiro j. Agora
divida 2003 por u: Temos que 2003 = q *u+r, onde 0=ru. Então,
fazendo m=r e n=qu, temos que f(r+f(qu)) = f(r)+f(qu), logo
f(r+qu) = f(r) + qu, logo f(2003) = f(r) + qu, ou seja, f(r) = 2003 - qu =
r, logo f(r) = r. Como r está na imagem de f, temos que r = 0, ou r
= u (este segundo caso não pode, pois r  u). Logo, u é divisor de
2003, portanto (como 2003 é primo), temos que u=1 ou u=2003.se u = 2003
(como é mínimo), temos que os valores de f são 0, 2003, 2003, ...,2003,
4006,4006, ..., 4006, 6009,... ou seja, f(m) = 2003*(1+parte
inteira[(m-1)/2003]).se u = 1, temos que f(m+1) = f(m) + 1, logo (por
indução), temos que f(m) = m.Isso dá o total de 3
funções.Abraços, Villard

- Mensagem Original De:
[EMAIL PROTECTED]Para: "[EMAIL PROTECTED]"
[EMAIL PROTECTED]Assunto: [obm-l] Eureka 18 e Olimpiada
CearenseData: 25/04/04 23:18 então parece que qualquer valor de k serve, mas f(1) =
2003, então temos 2004 valores para f(1), cada um determinando uma
função diferente. acho que é isso...opa, mas f(2003)
= 20032003 = q*k + r = f(2003) = f(q*k + r) = (q + r)k = r =
0 = k|2003então temos que tomar f(1) como divisor de
2003desculpem pelo erro bobo, espero que agora sim esteja
correto!=Instruções
para entrar na lista, sair da lista e usar a lista emhttp://www.mat.puc-rio.br/~nicolau/olimp/obm-l.html=

=
Instruções para entrar na lista, sair da lista e usar a lista em
http://www.mat.puc-rio.br/~nicolau/olimp/obm-l.html
=


Re: [obm-l] quadrado perfeito

2004-02-15 Por tôpico Villard
Se n=12, então a expressão é = 2^12(1+8+2^(n-12)) e temos
que 9 + 2^j = q^2, onde j=n-12. daí 2^j=(q-3)(q+3) e temos que q-3 e
q+3 são potências de 2 que diferem por 6 unidades, logo q-3=2 e q+3=8 e
temos que q=5 (isso dá j=4, ou seja, n=16, nesse caso o quadrado é
320^2).Se n12, então a expressão é 2^n*(1+2^(12-n)+2^(15-n)).Se
n for ímpar, então 2*(1+2^(12-n)+2^(15-n)) deve ser quadrado, isso só é
possível se o que estiver dentro do parêntesis for par, o que não ocorre
para n12.Logo n é par. Então 1+2^(12-n)+2^(15-n) deve ser quadrado,
ou seja, 1+2^(12-n)+2^(15-n) = 1+2^(12-n)*(1+2^3)=q^2, logo temos que q^2 -
(3*2^((12-n)/2))^2 = 1, o que não pode ocorrer, pois não temos nunca dois
quadrados sendo números consecutivos (veja que usei que n é
par.).Resposta: n=16. Abraços, Villard

- Mensagem Original De:
[EMAIL PROTECTED]Para: "[EMAIL PROTECTED]"
[EMAIL PROTECTED]Assunto: [obm-l] quadrado perfeitoData:
16/02/04 02:50Ola pessoal, Poderiam me dar um ajuda neste daqui ?
For what positive integer(s), n, is 2^12 + 2^15 + 2^n a perfect
square? 

=
Instruções para entrar na lista, sair da lista e usar a lista em
http://www.mat.puc-rio.br/~nicolau/olimp/obm-l.html
=


Re: [obm-l] 1o Dúvida

2004-01-27 Por tôpico Villard
Basta lembrar daquele critério de divisibilidade por 9: Um
número n e a soma dos seus dígitos S(n) deixam o mesmo resto na divisão por
9.Tente mostrar isso, como exercício.Então, por esse critério,
temos que n - S(n) é sempre múltiplo de 9. Logo, se n' é o inverso de n,
como S(n) = S( n' ), temos n - n' =(n-S(n))-(n'-S(n')) que é a diferença de
dois múltiplos de 9, o que dá um múltiplo de 9.Abraços,
Villard

- Mensagem Original De:
[EMAIL PROTECTED]Para: "[EMAIL PROTECTED]"
[EMAIL PROTECTED]Assunto: [obm-l] 1o DúvidaData: 27/01/04
15:48Essa é a minha primeira
dúvida aqui na lista.Por que todo número menos seu inverso(se é que
posso chamar assim) resultanum múltiplo de 9 ?Ex: 72-27; 47-74;
56-65; 32-23
etc._MSN
Hotmail, o maior webmail do Brasil. http://www.hotmail.com=Instruções
para entrar na lista, sair da lista e usar a lista emhttp://www.mat.puc-rio.br/~nicolau/olimp/obm-l.html=

=
Instruções para entrar na lista, sair da lista e usar a lista em
http://www.mat.puc-rio.br/~nicolau/olimp/obm-l.html
=


Re: Re:[obm-l] Resultado.

2003-12-15 Por tôpico Villard

 Bem, não sou a Nelly, mas posso tirar algumas de suas
dúvidas. Não sei como será a seleção esse ano, pois cada vez temos
mais pessoas querendo ir, mas o gugu me disse q isso será decidido e avisado
na semana olímpica.Em relação 'a
semana olímpica, parece que esse ano só será paga a viagem de quem foi ouro,
mas a estadia é paga para todos os medalhistas (me corrijam se eu estiver
errado). Em relação 'a imc, é bem provável que a sua universidade patrocine
(no meu caso, no ano passado, a ufrj patrocinou sem pensar duas vezes,
depois de o impa mandar uma cartinha milagrosa pro reitor).Você deve
estarem breve recebendo uma carta da secretaria da obm, avisando que
vc pode ir pra semanaolimpica, a qual vc vai ter q responder algumas
coisas... ela será realizada em belo horizonte, entre 16 e 23 de
dezembro.Espero ter ajudado, Um grande
abraço,VillardOi
Nelly e Carlos!Não dêem ouvido ao Stein, mandem só os prata e ouros
-- de outro jeito eunão vou conseguir ir, por que disputar com essa
gurizada cheia de medalhas,pra mim que sou fraquinho, vai ser difícil...
;) Deixando de lado abrincadeira, qual o critério de seleção para a IMC?
Há provas de seleção? Emque país será a próxima? Alguém financia os
alunos para irem ou eles tem quebuscar patrocínio/pagar por conta?
Quanto à Semana Olímpica, alguém dá ajudade custo para passagem e
hospedagem? Quando e onde vai ser?O quanto antes eu souber estas
respostas, melhor, pois posso passar, desdejá, a buscar financiamento da
minha universidade, caso ninguém dê ajuda
decusto.Abração,Duda.From: "Carlos"
[EMAIL PROTECTED] E ai Nelly! Fiquei
tão triste com meu resultado! :( Tem como saber minha pontuação de
cada questão, é porque achei que tinha feito tres questões (1, 4 e
5), inclusive mandei soluções pra lista obm-x e pelo visto tava
correto... De qualquer forma, vocês continuarão mandando apenas
prata e ouro, ou poderão abrir exceções, já que a IMC é por
universidade? Abraços,
Stein=Instruções
para entrar na lista, sair da lista e usar a lista emhttp://www.mat.puc-rio.br/~nicolau/olimp/obm-l.html=

=
Instruções para entrar na lista, sair da lista e usar a lista em
http://www.mat.puc-rio.br/~nicolau/olimp/obm-l.html
=


Re: [obm-l] alg-lin

2003-12-01 Por tôpico Villard
Você sempre tem um autovalor se considerar que seu espaço
vetorial é complexo, aí sim são as raízes de det(A-x*I)=0.E o problema
está errado... na verdade é "POSSUEM UM AUTOVETOR EM COMUM". Basta ver que
I*0=0*I e 0 e I não possuem autovalores em comum. Prova:Considere o
cunjunto U={v ; Sv=r*v} onder é um autovalor fixo de A. Veja que U
ésubespaço invariante por T,pois se v estáem U,
S(Tv)=T(Sv)=r*(Tv), logo Tv está em U. Então você pode considerar a
restrição de T a U (uma transformação T`:U -U) que possui
umautovetor vem U, tal que Tv=g*v e por definição de U, temos
Sv=r*v, logo possuem um autovetor em comum.Abraços, Villard

- Mensagem Original De:
[EMAIL PROTECTED]Para: "[EMAIL PROTECTED]"
[EMAIL PROTECTED]Assunto: Re: [obm-l] alg-linData:
01/12/03 21:43on 01.12.03 20:40,
Fabio Dias Moreira at [EMAIL PROTECTED]wrote:
On 12/01/03 13:12:42, Guilherme Carlos Moreira e Silva
wrote: Obrigado pela resposta. Foi muito
esclarecedora. Eu perguntei isto porque, numa prova que fiz,
havia a seguinte questão: Sejam T e S
duas transformações lineares tais que TS = ST. Prove que T e S
tem pelo menos um autovalor em comum. Na verdade
haviam dois itens, mas o primeiro não influencia o
segundo. Veja se estou certo ou errado. Se não posso garantir
que T ou S tem autovalor, como vou tentar provar que, além
disto, elas têm autovalor em comum?
[...] Toda transformação linear tem autovalores -- eles são
as raízes de det(A - xI) = 0; só que eles não são necessariamente
reais. []s,Eu tenho uma duvida conceitual. A
definicao de autovalor que o Fabio pareceestar usando acima eh a de raiz
do polinomio caracteristico do operadorcorrespondente. Mas e se tivermos
um operador sobre R^n cujo polinomiocaracteristico tem apenas raizes
complexas?Por exemplo, o operador T:R^2 - R^2 definido por T(x,y) =
(x+y,-x+y) temcomo polinomio caracteristico x^2 - 2x + 2, cujas raizes
sao 1+i e 1-i.1+i serah autovalor desse operador se existir algum vetor
nao nulo (a,b) deR^2 tal que T(a,b) = (1+i)*(a,b), mas isso eh
claramente impossivel. Entaoeh correto dizer que T nao tem autovalores?
Ou devemos dizer que osautovalores de T nao estao associados a nenhum
autovetor?Um
abraco,Claudio.=Instruções
para entrar na lista, sair da lista e usar a lista emhttp://www.mat.puc-rio.br/~nicolau/olimp/obm-l.html=

=
Instruções para entrar na lista, sair da lista e usar a lista em
http://www.mat.puc-rio.br/~nicolau/olimp/obm-l.html
=


Re:[obm-l] Olimpiada Iberoamericana-Universitária

2003-11-07 Por tôpico Villard
Olá, Cláudio. Essa eu posso responder pela Nelly. A Ibero
Universitária é um pouco diferente... todos podem competir... mas só podem
ser premiados os 10 melhores de cada país. Eu, por exemplo, vou fazer
:)AbraçosVillard

- Mensagem Original De:
[EMAIL PROTECTED]Para: "obm-l"
[EMAIL PROTECTED]Assunto: Re:[obm-l] Olimpiada
Iberoamericana-UniversitáriaData: 07/11/03 16:21

Oi, Nelly:

Quem sao os representantes do Brasil?

Um abraco,
Claudio.





De:
[EMAIL PROTECTED]




Para:
[EMAIL PROTECTED],
[EMAIL PROTECTED]




Cópia:





Data:
Fri, 07 Nov 2003 16:28:36
-0800




Assunto:
[obm-l] Olimpiada
Iberoamericana-Universitária






 Caros(as) amigos(as) das listas,
 
 Amanhã teremos a prova da VI Olimpíada Iberoamericana
 de Matemática Universitária.
 
 Por favor peço a todos os participantes para NÃO comentarem
 o conteúdo da prova em nenhuma lista de discussão nem por
outra
 via, isto porque trata-se de uma competião internacional e
há
 2 países que farão a prova numa data posterior.
 A prova poderá ser comentada a partir da divulgaão da mesma
no
 site da OBM. (eu avisarei).-
 
 Abraços a todos e boa sorte,
 
 Nelly.
 
 
 Instruões para entrar na lista, sair da lista e usar a lista
em
 http://www.mat.puc-rio.br/~nicolau/olimp/obm-l.html
 
 
 

=
Instruções para entrar na lista, sair da lista e usar a lista em
http://www.mat.puc-rio.br/~nicolau/olimp/obm-l.html
=


Re: [obm-l] Problema de soma.

2003-10-28 Por tôpico Villard
Veja que pra n ímpar, temos que
S(n)=1+(3-2)+(5-4)+...+(n-(n-1)) = 1+1+...+1=(número de ímpares de 1 até
n)=(n+1)/2.Portanto S(2003)/3 = 1002/3=667.Abraços,
Villard

- Mensagem Original De:
[EMAIL PROTECTED]Para: "[EMAIL PROTECTED]"
[EMAIL PROTECTED]Assunto: [obm-l] Problema de soma.Data:
28/10/03 23:11Se
Sn=1-2+3-4+...+(-1)^n-1*n ,para todo n inteiro e positivo,entãoS2003/3 é
igual a
?Obrigado,Carlos.=Instruções
para entrar na lista, sair da lista e usar a lista emhttp://www.mat.puc-rio.br/~nicolau/olimp/obm-l.html=

=
Instruções para entrar na lista, sair da lista e usar a lista em
http://www.mat.puc-rio.br/~nicolau/olimp/obm-l.html
=


Re: [obm-l] ciclo trigonométrico

2003-10-24 Por tôpico Villard
Basta você ver que A*sen(x) + B*cos(x) pode ser escrito como
R*sen(x+u), para algum par de constantes R e U.Isso é verdade, pq
tomando R e U tais que Rcos(u)=A e Rsen(u)=B ( e isso sempre é possível, com
R^2=A^2+B^2 e tg(u)=B/A ).Então, escreva 2*sen(x)+3*cos(x) = sqrt(13) *
sen(x+u) que tem máximo claramente em raiz de 13.Abraços, 
Villard

- Mensagem Original De:
[EMAIL PROTECTED]Para: "[EMAIL PROTECTED]"
[EMAIL PROTECTED]Assunto: [obm-l] ciclo
trigonométricoData: 24/10/03 13:47Qual o valor maximo da funcao f(x) = 3cos(x) +
2sen(x)?=Instruções
para entrar na lista, sair da lista e usar a lista emhttp://www.mat.puc-rio.br/~nicolau/olimp/obm-l.html=

=
Instruções para entrar na lista, sair da lista e usar a lista em
http://www.mat.puc-rio.br/~nicolau/olimp/obm-l.html
=


[obm-l] OBM-3, Questao 5

2003-10-23 Por tôpico Villard
Dei uma olhada na questão 5 da obm nivel 3 e achei
legal.Segue abaixo minha solução (e o tradicional espaço em branco pra
não atrapalhar que quer tentar
ainda)Minha
idéia é a seguinte : se a função é negativa pra algum valor de x, então ela
tb é para valores menores (pois é crescente). Então vou olhar para uma
sequência decrescente convergindo pra zero. Mas tenho que usar que é média
harmônica, logo, se 0 x  y, considere a sequência x(n)
definida por x(n) é a média harmônicade x(n+1) e y e x(1)=x. É fácil
ver então que x(n+1) = x(n)*y / (2y-x(n)) (claramente x(n)0), (por
construção a sequencia é claramente decrescente). Agora vamos usar a
hipótese do problema que f(média harmônica de a e b) = [f(a)+f(b) ]/ 2.
Temos que f(x(n)) = (1/2)*(f(x(n+1))+f(y)). Segue por indução então (ou
iterando a relação acima) que f(x(n)) = f(y) + 2^n (f(x)-f(y)) e como
f(x)-f(y)  0 e 2^n fica arbitrariamente grande para n grande, temos que
para n grande o lado direito dessa inequação é negativo, portanto temos
f(x(n)) negativo. Abraços. Villard

=
Instruções para entrar na lista, sair da lista e usar a lista em
http://www.mat.puc-rio.br/~nicolau/olimp/obm-l.html
=


Re: [obm-l] Ime...

2003-10-22 Por tôpico Villard
Hehe, Claudio, nem me lembro disso... Vou mostrarduas
soluções :1) Uma idéia legal é a seguinte : Se vc quer mostrar que I+A é
inversível, basta mostrar que o sistema linear homogêneo cuja matriz
principal é I+A é possível determinado, ou seja, não admite solução não
trivial. Suponha então por contradição que esse sistema possui uma solução
não trivial (x1,x2,...,xn) (represente por uma matriz coluna X n por 1 não
nula). Então (I+A)X=0, logo AX=-X. Agora use que A^3=kA
:-kX=kAX=A^3X=AAAX=AA(-X)=-AAX=-A(-X)=AX=-X então kX=X e como X
é não nula, k é igual a 1, uma contradição. Portanto o sistema
não possui solução não trivial, ou seja I+A é
inversível.2) Seja U=A+I, então temos A=U-I.
Agora use que A^3=kA : (U-I)^3=k(U-I), ou seja U^3-3*U^2+3*U-I = kU-kI,
logo temos que U*(U^2-3*U+(3-k)*I) = (1-k)*I, portanto U é ínversível ( sua
inversa é igual a [U^2-3*U+(3-k)*I ] / (1-k) (tá ok, pois k é
diferente de 1).Abraços, Villard

- Mensagem Original De:
[EMAIL PROTECTED]Para: "[EMAIL PROTECTED]"
[EMAIL PROTECTED]Assunto: Re: [obm-l] Ime...Data:
22/10/03 13:12on 22.10.03 12:26, [EMAIL PROTECTED] at
[EMAIL PROTECTED] wrote:
Acredito que esta questão já tenha
sido feita na listaSe alguém tiver paciência de repassa-la para
mimagradeço muito..Acho que estou atropelando os conceitos os
conceitos.Considere uma matriz A, nXn, de coeficientes reais, e k um
número real diferente de 1. Sabendo-se que A^3=k.A, prove que a matriz A+I é
invertível, onde I é a identidade de ordem n.
Vou usar um truquezinho que aprendi aqui na lista mesmo
(se nao me engano com o Villard).A ideia eh buscar uma inversa da
forma x*A^2 + y*A + z*I, onde x, y, z sao numeros reais a serem
determinados. (A + I)*(x*A^2 + y*A + z*I) = I =x*A^3 +
(x+y)*A^2 + (y+z)*A + (z-1)*I = 0 =(x+y)*A^2 + (y+z+k*x)*A + (z-1)*I
= 0.Agora eh soh igualar os coeficientes a zero.Fazendo z = 1,
cairemos no sistema:x + y = 0y + k*x = -1Solucao: x = 1/(1 -
k) e y = -1/(1 - k) (OK, pois k  1).Logo,
a matriz B = (1/(1-k))*A^2 - (1/(1-k))*A + I eh tal que (A+I)*B = I
=A + I eh inversivel.Um abraco,Claudio.


=
Instruções para entrar na lista, sair da lista e usar a lista em
http://www.mat.puc-rio.br/~nicolau/olimp/obm-l.html
=


Re: [obm-l] Ajuda (IME94)

2003-10-13 Por tôpico Villard
AMIQ é inscritível, logo DAC=QMI (soma dos ângulos opostos
igual a 180) ;ABCD é inscritível, logo DAC=DBC ;BMIN é inscritível,
logo DBC=NMI (soma dos ângulos opostos igual a 180).Logo, QMI=NMI, ou
seja, I pertence a bissetriz do ângulo M do quadrilátero MNPQ. Analogamente,
I pertence às bissetrizes dos ângulos N, P, Q do quadrilátero MNPQ, logo I
equidista dos lados deste quadrilátero, portanto I é o centro da
circunferência procurada.Abraços,  Villard

- Mensagem Original De:
[EMAIL PROTECTED]Para: "[EMAIL PROTECTED]"
[EMAIL PROTECTED]Assunto: [obm-l] Ajuda (IME94)Data:
13/10/03 12:48
Seja ABCD um quadrilátero convexo inscrito num círculo e seja I o ponto
de interseção de suas diagonais. As projeções ortogonais de I sobre os lados
AB , BC , CD e DA são , respectivamente , M , N , P e Q . Pove que o
quadrilátero MNPQ é inscritível a um círculo com centro em I.


Yahoo!
Mail - o melhor webmail do Brasil. Saiba
mais! 

=
Instruções para entrar na lista, sair da lista e usar a lista em
http://www.mat.puc-rio.br/~nicolau/olimp/obm-l.html
=


Re: [obm-l] Dúvida em equações polinomiais - As deduções estão erradas??? (Dúv ida muito suga!)

2003-09-26 Por tôpico Villard
Toda equação polinomial de grau ímpar possui pelo menos uma raiz REAL (e não
racional).Villard

- Mensagem Original De:
[EMAIL PROTECTED]Para: "[EMAIL PROTECTED]"
[EMAIL PROTECTED], "[EMAIL PROTECTED]"
[EMAIL PROTECTED]Assunto: [obm-l] Dúvida em equações
polinomiais - As deduções estão erradas??? (Dúv ida muito suga!)Data:
26/09/03 20:42Estou com uma
dúvida cruel em equações polinomiais e gostaria da ajudados
senhores.Consideremos uma equação polinomial de coeficientes
inteiros (A0, A1,A2, A3, ..., An).Sabe-se q toda equação tem um
número par de raízes complexas e um númeropar de raízes
irracionais.Logo, toda equação de grau ímpar terá ao menos uma raiz
racional.Sabe-se que todas as raízes racionais são da forma p/q tal
que:p e q são primos entre sip é divisor de Anq é divisor de
A0Consideremos a equação:3x^3 + 5x - 18 = 0É uma equação
de grau 3, logo terá ao menos uma raiz racional.Porém, traçando-se o
gráfico pelo Grafeq temos q há apenas uma raizreal, e esta raiz tem
valor aproximado 1,514735Esta raiz é única, portanto deveria ser
racional, não obedece à lei deformação p/q, portanto parece não ser
racional. A raiz multiplicada por3 deveria ser um número inteiro.O q
aconteceu afinal Onde está o erro Todas as raízes racionaissão
realmente da forma p/q (Caso isto seja falso, peço umademonstração
de que existem raízes racionais que não obedecem a esta leide formação,
pois eu tenho uma demonstração q afirma q as raízesobedecem a esta
lei)Aguardo respostas extremamente urgentes!!!Alexandre
Daibert - Juiz de
Fora=Instruções
para entrar na lista, sair da lista e usar a lista emhttp://www.mat.puc-rio.br/~nicolau/olimp/obm-l.html=

=
Instruções para entrar na lista, sair da lista e usar a lista em
http://www.mat.puc-rio.br/~nicolau/olimp/obm-l.html
=


Re: [obm-l] Resultados da Ibero 2003

2003-09-21 Por tôpico Villard
É, Morgado, lá na Romênia as condições não eram boas mesmo... um cheiro
bastante ruim. Deve ser legal participar de uma olimpíada na
Argentina,aproveitar o luxo deles e fazer a festa em cima dos hermanos
(hehe).Meus parabéns para você e para toda a equipe brasileira, David,
Samuel, Alex e Fábioobtiveram um excelente resultado.Um grande
abraço,Villard

- Mensagem Original De:
[EMAIL PROTECTED]Para: "[EMAIL PROTECTED]"
[EMAIL PROTECTED]Assunto: Re: [obm-l] Resultados da Ibero
2003Data: 21/09/03 10:01Voce
e o Villard, nossos herois que enfrentaram condiçoes precarias na Romenia
(parece que la nao havia nada parecido com a bailarina de vermelho), saibam
que os argentinos ocuparam para a Olimpiada dois dos tres melhores hoteis de
Mar del Plata. Acho que so nao ocuparam o melhor porque neste havia um
cassino.O banco deve permanecer em segredo por um ano, mas pode ser
usado em olimpiadas nacionais.Abraços.MorgadoEm Sun,
21 Sep 2003 09:06:42 -0300, "Marcio Afonso A. Cohen"
[EMAIL PROTECTED] disse: Meus parabéns ao
Fábio, ao Alex, ao Morgado e aos demais participantes da lisdta belo
resultado! Muito legal o fato de dois alunos fecharem a prova!
Parabéns! Abracos, Marcio PS: O banco da
Ibero tambem precisa ficar em segredo durante um certo tempo? Ou ele
pode ser divulgado
prontamente?=Instruções
para entrar na lista, sair da lista e usar a lista emhttp://www.mat.puc-rio.br/~nicolau/olimp/obm-l.html=

=
Instruções para entrar na lista, sair da lista e usar a lista em
http://www.mat.puc-rio.br/~nicolau/olimp/obm-l.html
=


Re: [obm-l] Questões da Olimpíada de Maio de 1999

2003-09-12 Por tôpico Villard
Sim, a resposta da primeira é essa mesmo.Pra segunda, basta olhar pra
soma dos elementos de cada linha.A soma da primeira e a soma da segunda
são 55. Se todos os últimos dígitos da terceira linha fossem distintos,
então a soma da terceira linha terminaria em 5, o que não é possível, pois
sua soma é 55+55=110.Abraços, Villard

- Mensagem Original De:
[EMAIL PROTECTED]Para: "OBM - Lista"
[EMAIL PROTECTED]Assunto: [obm-l] Questões da Olimpíada de
Maio de 1999Data: 10/09/03 15:22




Abaixo vão dois problemas da olimpíada de maio de 1999
que eu gostaria de saber as respostas:
Obs: O problema 1 eu resolvi e achei apenas 1 par de
tricúbicos consecutivos: 370 e 371. No entanto gostaria de confirmar se a
resposta é essa.

Problema 1
Um número natural de três algarismos é chamado de
tricúbico se é igual a soma
dos cubos dos seus dígitos. Encontre todos os pares de números consecutivos
tais que ambos sejam tricúbicos.

Problema 3
A primeira fileira da tabela abaixo se preenche
com os números de 1 a 10, em ordem crescente.






A segunda fileira se preenche com os números de 1
a 10, em qualquer ordem.
Em cada casa da terceira fileira se escreve a soma
dos dois números escritos nas casas acima.
Existe alguma maneira de preencher a segunda
fileira de modo que os algarismos das unidades dos números da terceira
fileira sejam todos distintos?


=
Instruções para entrar na lista, sair da lista e usar a lista em
http://www.mat.puc-rio.br/~nicolau/olimp/obm-l.html
=


Re: [obm-l] questões INVOCADAS!!!!! AJUDA!!!!!

2003-09-06 Por tôpico Villard
Bem, como há muito tempo não mando nada pra lista, vou ver se
respondo essas..1) Veja q se o grau de P é n,
então p(x)-p(x-1) tem grau n-1, logo n-1=2, então n=3. Veja que se
p(x)=ax^3+bx^2+cx+d, então p(x)-p(x-1)=3ax^2+(-3a+2b)x+(a-b+c) =
x^2Então a=1/3, b=1/2, c=1/6 e d é qualquer.2) Veja que i^2 = p(i)-p(i-1), logo
S=p(n)-p(0)=(1/3)n^3+(1/2)n^2+(1/6)n3) Eu acho
que vc se confundiu... essa raiz cúbica q tá dentro da outra raiz cúbica
deve ser raiz quadrada.Se for assim temos x^3=4+3*raiz_cúbica[8/27]*x,
logo x^3-2x-4=0. Temos que x=2 é raiz, logo vamos fatorar x-2 :
x^3-4x+2x-4=0, logo x(x^2-4)+2(x-2)=0... (x-2)(x^2+2x+2)=0. Como vc quer x
real, x=2.4) Se z=512, então seu módulo é 512,
logo |2^n(1-i)^n| = 512, logo 2^n * |1-i|^n=512, ou seja 2^((3n)/2)=512=2^9,
portanto n=6. Mas vc deve testar se n= 6 satisfaz : (2-2i)^6 = 2^6*(-2i)^3 =
2^6*2^3, ok !5) A soma dos coeficientes é P(1) =
(-1)^36=16) Os números 1,2,3,4,5 são as raízes
de P(x)-1, logo P(x)-1 = A(x-1)(x-2)(x-3)(x-4)(x-5). Como P(6)=0, temos
A=-1/120, logo, P(0)-1=(-1/120)*(-120), logo P(0)=2Abraços, Villard

- Mensagem Original De:
[EMAIL PROTECTED]Para: "[EMAIL PROTECTED]"
[EMAIL PROTECTED]Assunto: [obm-l] questões INVOCADAS!
AJUDA!Data: 06/09/03 15:44olá. olhem algum probleminhas chatos!!11) Determine os
polimônios P do terceiro grau que, para todo número real x,se tenha
P(x)-P(x-1) = x^22) usando o resultado da parte a, calcule, em
função de n:S = {E} i variando de 1 até ni^2 = 1^2 + 2^2 + 3^2 +
... n^23) Mostrar que é inteiro o númeroraiz cúbica [2 +
10/9 * raiz cúbica(3)] + raiz cúbica (2 - 10/9 *
raizcúbica(3)]observação = x é multiplicaçãoraiz cúbica de tudo
que estar dentro do colchetes4) seja um número complexo Z =
(2-2i)^n , onde n pertence aos naturaisdiferente de zero. se Z= 512
então n vale quanto ?5) se P(x)= AnX^n + An-1X^(n-1) + ... + A1x
+ Ao é um polinômio. An + An-1 +... A1 + Ao é a soma dos coeficientes do
polinômios P(x). A soma doscoeficientes do polinômio (4x³ - 2x² -2x
-1)^36 éa) 1 b)0 c)-36 d)-1 e) não sei, pois só burro!6) SE P(x)
é um polinômio do 5º grau que satifaz as
confições1=P(1)=P(2)=p(3)=P(4)=P(5) e P(6)=0 , então temos:a)
P(0)=4 b)P(0)=9 c)P(O)=3 d)P(0)=2 e) não sei, pois sou
burro!_Voce
quer um iGMail protegido contra vírus e spams?Clique aqui: http://www.igmailseguro.ig.com.brOfertas imperdíveis!
Link: http://www.americanas.com.br/ig/=Instruções
para entrar na lista, sair da lista e usar a lista emhttp://www.mat.puc-rio.br/~nicolau/olimp/obm-l.html=

=
Instruções para entrar na lista, sair da lista e usar a lista em
http://www.mat.puc-rio.br/~nicolau/olimp/obm-l.html
=


Re: [obm-l] Mais indução...

2003-09-06 Por tôpico Villard
É fácil...Faça T(n) = (2n)!*(n+1)/[ (n!)^2 * 4^n]. O que
vc quer provar é que T(n)  1. Agora calcule T(n+1) em função de T(n)
:T(n+1) = (2(n+1))!*(n+2)/[ ((n+1)!)^2 * 4^(n+1)] = (2n+1)(n+2) *
T(n)/2(n+1)^2. Agora veja que (2n+1)(n+2) 2(n+1)^2, pois isso é
equivalente a 2n^2+5n+22n^2+4n+2, que é verdade pois n0. Logo
T(n+1)T(n). Como T(1)=1, a indução está
feita.Villard

=
Instruções para entrar na lista, sair da lista e usar a lista em
http://www.mat.puc-rio.br/~nicolau/olimp/obm-l.html
=


Re: [obm-l] Para Villard

2003-08-03 Por tôpico Rodrigo Villard Milet
Acho que você está se confundindo... quem falou isso foi o Morgado. Eu
apenas respondi a resposta, sem fazer nenhuma menção a esse senhor.
Abraços
 Villard
-Mensagem original-
De: alex.rabelo1989 [EMAIL PROTECTED]
Para: [EMAIL PROTECTED] [EMAIL PROTECTED]
Data: Sábado, 2 de Agosto de 2003 20:46
Assunto: Re: [obm-l] Para Villard


 Villard você poderia especificar quem é o tal senhor
obeso ao qual você se refere no e-mail enviado dia
02/08/2003.

Antecipadamente agradecido,
Alex Rabelo.


__
Acabe com aquelas janelinhas que pulam na sua tela.
AntiPop-up UOL - É grátis!
http://antipopup.uol.com.br/


=
Instruções para entrar na lista, sair da lista e usar a lista em
http://www.mat.puc-rio.br/~nicolau/olimp/obm-l.html
=


=
Instruções para entrar na lista, sair da lista e usar a lista em
http://www.mat.puc-rio.br/~nicolau/olimp/obm-l.html
=


Re: [obm-l] Jornal Nacional

2003-08-02 Por tôpico Villard
Olá, muito obrigado pela citação.Mas só uma correção... era minha avó,
não minha mãe :)Um Grande abraço, Villard

- Mensagem Original De:
[EMAIL PROTECTED]Para: "[EMAIL PROTECTED]"
[EMAIL PROTECTED]Assunto: [obm-l] Jornal NacionalData:
01/08/03 21:14Para quem nao
assistiu ao Jornal Nacional de hoje:Foi noticiado o desempenho dos
cariocas (houve bastante bairrismo: osnao-cariocas foram ignorados) com
direito a entrevista do Villard, damae dele, do Marcio Cohen, do pai
dele e do Luciano. Dessa vez as coisasforam bem feitas, ninguem pediu
que eles fizessem uma conta grande, comofizeram com o Rui, no
Fantastico, quando ele ganhou ouro na IMO. Paraquem ve, as vezes, os
"matematicos" que frequentam o programa de umcerto senhor obeso
(considerado muito inteligente porque fala tantosidiomas quanto o
Romario), sempre com formulas magicas para ganhar naloteria ou com
metodos magicos para fazer contas mais depressa, semprede nivel
intelectual baixissimo (o que a concordancia sofre na bocadesse
individuos!) e querendo vender livros pessimos, errados e inuteis,foi um
progresso
gigantesco.=Instruções
para entrar na lista, sair da lista e usar a lista emhttp://www.mat.puc-rio.br/~nicolau/olimp/obm-l.html=

=
Instruções para entrar na lista, sair da lista e usar a lista em
http://www.mat.puc-rio.br/~nicolau/olimp/obm-l.html
=


Re: [obm-l] Curvas

2003-07-21 Por tôpico Villard
Batize-as como "Curvas de Estanislau"

- Mensagem Original De:
[EMAIL PROTECTED]Para: "[EMAIL PROTECTED]"
[EMAIL PROTECTED]Assunto: [obm-l] CurvasData: 21/07/03
11:09




 Caros amigos, as curvas abaixo possuem algum nome
especial? Como elas são feitas?

 Desde já agradeço!

 Davidson Estanislau

B

=
Instruções para entrar na lista, sair da lista e usar a lista em
http://www.mat.puc-rio.br/~nicolau/olimp/obm-l.html
=


Re: [obm-l] problemas propostos ...

2003-07-21 Por tôpico Villard
1) Seja A = {a(1),...,a(n)} o seu conjunto. Considere os números a(1),
a(1)+a(2), ..., a(1)+a(2)+...+a(n). Se um deles for múltiplo de n acabou.
Caso contrário, temos n números e n-1 restos possíveis na divisão por n
(1,2,..,n-1). Pelo princípio das gavetas, temos que dois deles deixam o
mesmo resto na div por n. Qd vc pega a diferença deles, vc tem o
resultado.2) Veja que pela equação dada, dentre x e y, um deles deve
ser =0, pois 1+x^2 =0 e sqrt(1-4y^2) - 1 =0. Logo xy=0.
Basta ver então que o xy=0 pode ser atingido, fazendo y=0 e x igual a
qq coisa... veja se o enunciado é esse mesmo.Abraços,
Villard

- Mensagem Original De:
[EMAIL PROTECTED]Para: "[EMAIL PROTECTED]"
[EMAIL PROTECTED]Assunto: [obm-l] problemas propostos
...Data: 20/07/03 21:45Não estou conseguindo enxergar o
princípio das gavetas nesse exercicio do eureka 11; fui analisando todas as
possibilidades para os subconjuntos de inteiros, mas não chego a concluão
alguma. Deve ser simples, mas não vejo1) Mostre que em qualquer
coleão de n inteiros há um subconjunto cuja soma dos elementos é divisível
por n.2) Determine o valor máximo do produto xy se os números reais x e
y satisfazem a relaão: y(1+x^2)=x(sqrt(1-4y^2)-1). Qualquer
ajuda nesses exercícios eu
agradeço.
Korshinói 

=
Instruções para entrar na lista, sair da lista e usar a lista em
http://www.mat.puc-rio.br/~nicolau/olimp/obm-l.html
=


Re: [obm-l] Números binomiais

2003-07-19 Por tôpico Villard
Basta você ver,usando o binômio denewton,que isso é a parte real
de (1+i)^4n. E como (1+i)^4n = (-4)^n q é real, sua soma vale
(-4)^n.Abraços, Villard

- Mensagem Original De:
[EMAIL PROTECTED]Para: "[EMAIL PROTECTED]"
[EMAIL PROTECTED]Assunto: [obm-l] Números binomiaisData:
19/07/03 23:49
Como calcular o somatório de...
Notando (a/b) como binomial

1 - (4n/2) + (4n/4)-...-(4n/ 4n -2) + 1

...?
 


Yahoo! Mail Mais espaço,
mais segurança e gratuito: caixa postal de 6MB, antivírus, proteção contra
spam. 

=
Instruções para entrar na lista, sair da lista e usar a lista em
http://www.mat.puc-rio.br/~nicolau/olimp/obm-l.html
=


[obm-l] Problemas IMO - Questao 4

2003-07-15 Por tôpico Rodrigo Villard Milet
Parece estar certo... Eu fiz uma solução legalzinha... segue :



Deixo um espaço em branco...





.








.

















Veja que podemos supor que P está fora de AB e Q está dentro de BC, pois
como A+C= 180, um dos A ou C deve ser agudo e o outro obtuso. [XYZ] = área
do triângulo XYZ.
Veja que [ACQ]=[APC] (pois como PR=QR, então [APR]=[ARQ] e [CPR]=[CRQ], já
que P,Q e R são colineares - reta de Simson)
Logo [BCP]-[ABC]=[ABC]-[ABQ], então BP.BC-BA.BC=BA.BC-BA.BQ, o que é
equivalente a BA.(BC-BQ) = BC.(BP-BA) ... BA.CQ=BC.PA. Como PA/CQ =
AD.cosC/CD.cosC  = AD/CD, segue que BA/BC=AD/CD, o que finaliza o problema.

Abraços,
 Villard
-Mensagem original-
De: [EMAIL PROTECTED] [EMAIL PROTECTED]
Para: [EMAIL PROTECTED] [EMAIL PROTECTED]
Data: Terça-feira, 15 de Julho de 2003 13:00
Assunto: [obm-l] Problemas IMO - Questao 4


So um pequeno detalhe... nao precisei usar o fato de ABCD ser incritivel
(pelo menos nao explicitamente). Alguem poderia comentar isso?

#
# MSc. Edson Ricardo de A. Silva#
# Computer Graphics Group (CRAB)#
# Federal University of Ceara (UFC) #
#

 achei legal essa sua solucao por complexos. Uma outra solucao
 trivial (e acho que a de 99% dos participantes) seria a seguinte:

 quad. APDR inscritivel  =  PR = AD.sen(BAC)
 quad. CQRD inscritivel  =  RQ = DC.sen(ACB)

 PR = RQ  =  AD/DC = sen(ACB)/sen(BAC) = AB/BC  (lei dos senos)   (*)

 Sendo S e T os pontos de interseccao das bissetrizes internas dos
 angulos ABC e ADC, respectivamente, com o lado AC, temos:

 AS/SC  =  AB/BC  =  AD/DC  =  AT/TC   Logo, S = T
   (1)   (2)   (3)

 (1) e (3) - teorema da bissetriz interna
 (2) - por (*)

 abracos,
=
Instruções para entrar na lista, sair da lista e usar a lista em
http://www.mat.puc-rio.br/~nicolau/olimp/obm-l.html
=


=
Instruções para entrar na lista, sair da lista e usar a lista em
http://www.mat.puc-rio.br/~nicolau/olimp/obm-l.html
=


Re: [obm-l] Sugestao para solucao

2003-07-06 Por tôpico Rodrigo Villard Milet
Olhem o que eu escrevi no meio da msg
-Mensagem original-
De: Domingos Jr. [EMAIL PROTECTED]
Para: [EMAIL PROTECTED] [EMAIL PROTECTED]
Data: Domingo, 6 de Julho de 2003 23:57
Assunto: Re: [obm-l] Sugestao para solucao


1) Seja A um anel, tal que x^2 = x para todo x de A. Prove que A eh
comutativo.
A minha tentativa foi a seguinte: Tomei x e y de A. Assim, (x + y)^2 = x +
y.
Desenvolvendo, temos:
x.x + x.y + y.x + y.y  = x + y.
x^2 + x.y + y.x  +  y^2 = x + y.
Apos a simplificacoes possiveis, cheguei a
xy = -(yx)
Mas isso nao significa que A eh comutativo. Onde errei?

que tal:
-(yx) pertence a A, então
-(yx) = [-(yx)]² = (yx)² = yx

Aqui você não pode fazer isso :  [-(yx)]² = (yx)² , pois [-(yx)]²
=(-yx)*(-yx) e vc ñ sabe ainda q x e y comutam...  o seu argumento abaixo
mostra que (-1)^2 = 1 e como -1 está em A, temos que 1=(-1)^2=-1, portanto
xy = -yx = (-1)*yx = 1*yx = yx

para ver que (-a)² = a², temos
0 = a.0 = a.(a - a) = a² + a(-a) = a.(-a) = -a²
da mesma forma
0 = 0.a = (a - a).a = a² + (-a).a = (-a).a = -a²
tb temos:
(a - a)² = 0
a² + a(-a) + (-a).a + (-a)² = 0
a² - a² - a²+ (-a)² = 0
- a²+ (-a)² = 0 = (-a)² = a²

acho que nem precisava dessa última parte, mas serve como curiosidade...

Abraços,
 Villard
=
Instruções para entrar na lista, sair da lista e usar a lista em
http://www.mat.puc-rio.br/~nicolau/olimp/obm-l.html
=


=
Instruções para entrar na lista, sair da lista e usar a lista em
http://www.mat.puc-rio.br/~nicolau/olimp/obm-l.html
=


Re: [obm-l] Numeros complexos

2003-06-06 Por tôpico Rodrigo Villard Milet



Temos que |w|= -w^2. Tire módulo dos dois lados : ||w|| = |-w^2|, logo 
|w|=|w|^2, ou seja, |w| é0 ou 1.No primeiro caso, w=0.Retorne à 
equação original, |w|=1 implica w^2 + 1 = 0, logo w=+-i, que claramente 
satisfazem a equação.
Abraços,
Villard

  -Mensagem original-De: 
  Oswaldo Stanziola [EMAIL PROTECTED]Para: 
  [EMAIL PROTECTED] [EMAIL PROTECTED]Data: 
  Sexta-feira, 6 de Junho de 2003 19:40Assunto: [obm-l] Numeros 
  complexos
  Boa noite pessoal.
  
  Por gentileza, gostaria de uma ajuda na resolucao 
  do exercicio:- Determine os valores de w que satisfazem
  a igualdadew ^2 + | w | = 0, onde |w| eh o 
  modulo do
  numero complexo w.
  
  Resp. em um livro a resposta eh: 1, i e 
  -i
   em outro: 1 e 
  i
  Obrigado.
  Oswaldo


Re: [obm-l] primos

2003-03-09 Por tôpico Rodrigo Villard Milet



Qualquer n composto serve.
Villard

  -Mensagem original-De: 
  [EMAIL PROTECTED] [EMAIL PROTECTED]Para: [EMAIL PROTECTED] [EMAIL PROTECTED]Data: 
  Domingo, 9 de Março de 2003 23:17Assunto: [obm-l] 
  primosMe apontem um primo n que torna 2 ^ n - 
  1 um inteiro composto .Grato!! 



Re: [obm-l] primos

2003-03-09 Por tôpico Rodrigo Villard Milet



Desculpe pela mensagem anterior... não tinha visto o "primo"
Essa pergunta é pertinente, pois a gente sabe o q eu mandei na msg 
anterior... ou seja, q n composto implica 2^n - 1 composto. 2^11 - 1 = 23*89 
mostra q a recíproca é falsa.
Abraços, 
Villard

  -Mensagem original-De: 
  Rodrigo Villard Milet [EMAIL PROTECTED]Para: 
  [EMAIL PROTECTED] [EMAIL PROTECTED]Data: 
  Domingo, 9 de Março de 2003 23:45Assunto: Re: [obm-l] 
  primos
  Qualquer n composto serve.
  Villard
  
-Mensagem original-De: 
[EMAIL PROTECTED] [EMAIL PROTECTED]Para: [EMAIL PROTECTED] [EMAIL PROTECTED]Data: 
Domingo, 9 de Março de 2003 23:17Assunto: [obm-l] 
primosMe apontem um primo n que torna 2 ^ 
n - 1 um inteiro composto .Grato!! 
  


Re: [obm-l] Problema simples...

2003-03-06 Por tôpico Rodrigo Villard Milet



Realmente é simples... para a letra c, note que se A tem posto 1, então 
posso escrever A=u.(vT). Isto acontece pq as colunas de A são múltiplas (por 
exemplo) da primeira coluna, daí segue. Então, como A^2=m*A, com m = (uT).v, 
temos que A^r=m^(r-1) * A. Então, se você quer achar a inversa de I-A, vale 
lembrar da identidade de números reais : (1+x+...+x^k)(1-x)=1-x^(k+1), em 
particular se módulo de x é 1 e se k tende a infinito, temos (1-x)^(-1) 
=1+x+x^2+...
Portanto, cabe tentarmos a inversa do tipo " I+A+A^2+..." (coloquei entre 
aspas, pois essa soma ainda ñ faz sentido).
Temos " I+A+A^2+... = I+A+m*A+m^2 * A +... =I+A*(1+m+m^2+...)". Se m 
tiver módulo 1, então o q está em aspas é B = I + A*(1/(1-m)). 
Então o que vc faz para o caso geral ? Basta verificar que B é inversa de A 
em qualquer caso, pois :
AB=A[I + A*(1/(1-m))]=A+ A^2 (1/(1-m)) = A+m*A*(1/(1-m))= A*[1/(1-m)] = 
B-I, logo B(I-A)=I, portanto (I-A)B=I, logo B é a inversa de A, caso m seja 
diferente de 1, já que aparece 1-m no denominador.

Se m=1, então A^2=A, logo (I-A)A=0. Suponha que I-A tenha uma inversa B. 
Portanto B(I-A)A=B*0=0, logo A=0, já que B(I-A)=I, mas nesse caso o posto de A é 
0 e não 1, uma contradição.

Isso finaliza o problema.

Uma outra maneira de achar a inversa é vc procurar uma inversa da forma 
B=p*I+q*A, já que as potências mais altas de A são reduzidasà A.
Daí não fica difícil escolher p e q : 
B(I-A)=(pI+qA)(I-A)=pI+(q-p)A-qA^2=pI+(q-qm-p)A. Então escolha p=1 e q-qm-p=0, 
ou seja q=1/(1-m), o que nos dá a mesma conclusão q antes.

Abraços, Villard

  -Mensagem original-De: 
  leandro [EMAIL PROTECTED]Para: 
  [EMAIL PROTECTED] [EMAIL PROTECTED]Data: 
  Terça-feira, 4 de Março de 2003 21:42Assunto: [obm-l] Problema 
  simples...
  
  Amigos,
  
  Esse e um problema simples, mas eu 
  nao consegui ver a solucao da parte c:
  
  
  Seja u,v vetores em R^n e 
  A=uvT. Entao, mostre que
  
  
  (a) 
  A^2 = (u.v) A. Esse eu fiz. 
  (u.v denota o produto interno)
  (b) 
  Use a parte (a) para mostrar que 
  se u.v e diferente de zero, entao (u.v) e o unico autovalor diferente de zero 
  de A. (Esse eu fiz)
  (c) 
  Use a parte (a) e a parte (b) para 
  mostrar que se A tem posto 1, entao I-A e inversivel se e somente se A^2 e 
  diferente de A. 
  
  
  So nao consegui ver a parte (c). 
  Alguem pode me ajudar. 
  
  
  Leandro. 



Re: [obm-l] Problema

2003-01-13 Por tôpico Rodrigo Villard Milet
Se você sabe um pouco de álgebra linear fica fácil. Seja k o menor natural
tal que A^k = 0. Portanto, existe um vetor v de R^n tal que A^(k-1) * v não
é zero. Agora provamos que v, Av, ... , A^(k-1) * v são um conjunto l.i.
Suponha que temos a(0)*v + a(1)*Av + ... + a(k-1)*A^(k-1) * v = 0, com a(i)
reais. Multiplique essa equação por A^(k-1) à esquerda, daí segue que
a(0)=0. Depois multiplique por A^(k-2) e terá que a(1)=0. Dessa mesma forma,
mostramos que a(0)=a(1)=...=a(k-1)=0, logo o conjunto é l.i. Isso prova em
particular que k=n, pois não podemos ter mais de n vetores l.i em R^n.
Voltando ao seu problema se temos que A^(n+1)=0 é pq n+1 não pode ser o
menor número k tal que A^k=0 (pelo OBS acima). Então k=n. Se k=n, acabou,
se k é menor que n, segue que A^n = A^k * A^(n-k) = 0.
Talvez tenha um jeito mais simples pra fazer isso..
Abraços,
 Villard

-Mensagem original-
De: [EMAIL PROTECTED] [EMAIL PROTECTED]
Para: [EMAIL PROTECTED] [EMAIL PROTECTED]
Data: Segunda-feira, 13 de Janeiro de 2003 18:34
Assunto: [obm-l] Problema



OLa galera,

Estou enviando um bom problema de matriz. La vai...
Seja  A uma matriz nxn. Prove que se A^(n+1) = 0, então A^n = 0.

Cícero Thiago





--
Use o melhor sistema de busca da Internet
Radar UOL - http://www.radaruol.com.br



=
Instruções para entrar na lista, sair da lista e usar a lista em
http://www.mat.puc-rio.br/~nicolau/olimp/obm-l.html
O administrador desta lista é [EMAIL PROTECTED]
=


=
Instruções para entrar na lista, sair da lista e usar a lista em
http://www.mat.puc-rio.br/~nicolau/olimp/obm-l.html
O administrador desta lista é [EMAIL PROTECTED]
=



[obm-l] Re: [obm-l] NOVO MEMBRO E UMA DÚVIDA

2002-12-20 Por tôpico Rodrigo Villard Milet
Estude pelas eurekas e pelo site www.kalva.demon.co.uk . Esse site, que já
foi mencionado diversas vezes aqui na lista, possui uma quantidade absurda
de provas ( a maioria com suloções ).

Abraços,
 Villard
-Mensagem original-
De: Helder Oliveira de Castro [EMAIL PROTECTED]
Para: [EMAIL PROTECTED] [EMAIL PROTECTED]
Data: Quinta-feira, 19 de Dezembro de 2002 00:15
Assunto: [obm-l] NOVO MEMBRO E UMA DÚVIDA


Muito obrigado Villard, pela solução. Achei bastante interessante o
raciocínio. O problema era aparentemente dificílimo, mas essa idéia
esclareceu tudo (até reparei que os valores davam certinho). Você falou que
o Antonio Munhoz é aluno seu, certo? Eu queria saber então se você (ou
alguém da lista) não tem algum material bom para treinar para a OBM,
principalmente este tipo de raciocínio. É por isso que entrei para a lista
e
também compro as revistas Eureka!'s.

Valeus,
   Helder

_
Voce quer um iGMail protegido contra vírus e spams?
Clique aqui: http://www.igmailseguro.ig.com.br

=
Instruções para entrar na lista, sair da lista e usar a lista em
http://www.mat.puc-rio.br/~nicolau/olimp/obm-l.html
O administrador desta lista é [EMAIL PROTECTED]
=


=
Instruções para entrar na lista, sair da lista e usar a lista em
http://www.mat.puc-rio.br/~nicolau/olimp/obm-l.html
O administrador desta lista é [EMAIL PROTECTED]
=



[obm-l] Re: [obm-l] Re:_[obm-l]_Re:_[obm-l]_RAIZ_CÚBICA_DE_7

2002-12-18 Por tôpico Rodrigo Villard Milet
A minha última msg mostra que é irracional. Basta ver que não é inteiro,
situando o seu primo entre duas n-ésimas potências.

Abraços,
 Villard
-Mensagem original-
De: Carlos Maçaranduba [EMAIL PROTECTED]
Para: [EMAIL PROTECTED] [EMAIL PROTECTED]
Data: Quarta-feira, 18 de Dezembro de 2002 14:54
Assunto: Re: [obm-l] Re:_[obm-l]_Re:_[obm-l]_RAIZ_CÚBICA_DE_7


Como seria entao a raiz n-ésinma de um número primo qualquer?

___
Busca Yahoo!
O melhor lugar para encontrar tudo o que você procura na Internet
http://br.busca.yahoo.com/
=
Instruções para entrar na lista, sair da lista e usar a lista em
http://www.mat.puc-rio.br/~nicolau/olimp/obm-l.html
O administrador desta lista é [EMAIL PROTECTED]
=


=
Instruções para entrar na lista, sair da lista e usar a lista em
http://www.mat.puc-rio.br/~nicolau/olimp/obm-l.html
O administrador desta lista é [EMAIL PROTECTED]
=



[obm-l] Re: [obm-l] Re: [obm-l] RAIZ CÚBICA DE 7

2002-12-17 Por tôpico Rodrigo Villard Milet
Na verdade isso é muito mais geral. Se raiz n-ésima de a^m (a natural) não é
inteiro, então deve ser irracional. É fácil provar isso, se vc sabe um
critério para achar raízes racionais de equações com coeficientes inteiros.
LEMA: Dada a equação A(n)x^n + A(n-1)x^(n-1) +... +A(1)x+A(0)=0 e p/q (na
forma irredutível) é raiz, então p divide A(0) e q divide A(n).
Prova: Substitua p/q na equação. Então A(n)p^n = -q*[A(n-1)p^(n-1)
+...+A(0)q^(n-1)] e como p e q não tem fatores em comum, segue que todos os
fatores de q se encontram em A(n), logo q | A(n). Analogamente p | A(0).

Então considere a equação x^n - a^m=0. Temos que  raiz n-ésima de a^m é
raiz. Então, pelo lema, se é racional (p/q), teríamos p | a^m e q | 1, logo
p/q é inteiro, o que é uma contradição, já que estamos supondo que não é
inteiro.
Logo raiz n-ésima de a^m (a natural ), se não é inteiro, é irracional.

Abraços,
 Villard




-Mensagem original-
De: Cláudio (Prática) [EMAIL PROTECTED]
Para: [EMAIL PROTECTED] [EMAIL PROTECTED]
Data: Terça-feira, 17 de Dezembro de 2002 18:04
Assunto: [obm-l] Re: [obm-l] RAIZ CÚBICA DE 7


A demonstração segue a mesma lógica:

7^(1/3) = m/n  com mdc(m,n) = 1
7 = (m^3) / (n^3)
m^3 = 7 * (n^3)
m^3 é múltiplo de 7
m é múltiplo de 7
m^3 é múltiplo de 7^3 = 343
m^3 = 343 * k

Mas, neste caso, 343 * k = 7 * (n^3) (ambos são iguais a m^3), ou seja:
7 * (7*k) = n^3
n^3 é múltiplo de 7
n é múltiplo de 7 == contradição, pois 7 divide m e mdc(m,n) = 1

Na verdade, o mesmo tipo de demonstração se aplica com qualquer número
primo
(não apenas o 7) e qualquer expoente (não apenas o 3).

O ponto crucial é a inferência m^3 é múltiplo de 7 == m é múltiplo de 7,
que só é verdadeira porque 7 é primo.

Um abraço,
Claudio Buffara.

- Original Message -
From: JOÃO CARLOS PAREDE [EMAIL PROTECTED]
To: OBM [EMAIL PROTECTED]
Sent: Tuesday, December 17, 2002 4:27 PM
Subject: [obm-l] RAIZ CÚBICA DE 7


Em livros sobre conjuntos numéricos, eles quase sempre
apresentam uma prova por absurdo da irracionalidade da
raiz quadrada de 2:

sqrt(2)=p/q, sendo mdc(p,q)=1
2=(p*p)/(q*q)
2*q*q=p*p
Com isto p é par.
Analogamente se prova que q é par, caindo no absurdo.

Mas, por exemplo, com raiz cúbica de 7, como faço?

=

 JOÃO CARLOS PAREDE


___
Busca Yahoo!
O melhor lugar para encontrar tudo o que você procura na Internet
http://br.busca.yahoo.com/
=
Instruções para entrar na lista, sair da lista e usar a lista em
http://www.mat.puc-rio.br/~nicolau/olimp/obm-l.html
O administrador desta lista é [EMAIL PROTECTED]
=

=
Instruções para entrar na lista, sair da lista e usar a lista em
http://www.mat.puc-rio.br/~nicolau/olimp/obm-l.html
O administrador desta lista é [EMAIL PROTECTED]
=


=
Instruções para entrar na lista, sair da lista e usar a lista em
http://www.mat.puc-rio.br/~nicolau/olimp/obm-l.html
O administrador desta lista é [EMAIL PROTECTED]
=



[obm-l] Re: [obm-l] identidades algébricas nos complexos

2002-12-16 Por tôpico Rodrigo Villard Milet
Você pode tornar tudo isso mais preciso...
O teorema é  Se duas funções coincidem num conjunto que possui um ponto de
acumulação ( por exemplo, um intervalo, como vc disse ), então elas
coincidem .
Basta mostrar que os zeros de funções holomorfas não identicamente nulas são
isolados.
Dada f holomorfa com zero em z0, é fácil ver que não podem se anular todas
as derivadas de f em z0, pois f seria nula. Então seja m a ordem da primeira
derivada não nula de f em z0. Segue que f(z) = (z-z0)^m * h(z), com h(z0)
não nulo. Agora é fácil, numa vizinhança de z0, (z-z0)^m é diferente de 0
para z0 diferente de 0. E além disso h(z) é diferente de zero, pois é
continua. Logo nessa vizinhança só há o zero z0.
Agora considere duas funções f e g que coincidem num conjunto com ponto de
acumulação a.  Seja h=f-g. Temos que h é nula num conjunto com ponto de
acumulação, logo possui zero não isolado, uma contradição, logo h é
identicamente nula , donde f==g.

Qualquer coisa, pergunte de novo..
Abraços, Villard

-Mensagem original-
De: Domingos Jr. [EMAIL PROTECTED]
Para: [EMAIL PROTECTED] [EMAIL PROTECTED]
Data: Segunda-feira, 16 de Dezembro de 2002 18:51
Assunto: [obm-l] identidades algébricas nos complexos


Olá a todos!

Existe um teorema que afirma que se uma função complexa definida num aberto
que possui um intervalo da reta real e neste intervalo existe uma
identidade
algébrica envolvendo essa função, então a identidade também é válida no
domínio complexo.

É um teorema muito útil para sabermos que identidades como sen²z + cos²z =
1
valem pra z complexo.

Alguém sabe como demonstrar esse teorema?

[ ]'s

=
Instruções para entrar na lista, sair da lista e usar a lista em
http://www.mat.puc-rio.br/~nicolau/olimp/obm-l.html
O administrador desta lista é [EMAIL PROTECTED]
=


=
Instruções para entrar na lista, sair da lista e usar a lista em
http://www.mat.puc-rio.br/~nicolau/olimp/obm-l.html
O administrador desta lista é [EMAIL PROTECTED]
=



[obm-l] Re: [obm-l] NOVO MEMBRO E UMA DÚVIDA

2002-12-15 Por tôpico Rodrigo Villard Milet
Segue abaixo a solução do problema 5 da olimpíada do nível 3. (É +- a
solução dada por um aluno meu, o Antônio Munhoz, que foi prata).
Só pra relembrar o enunciado :
Temos um número finito de quadrados, de área total 4. Prove que é possível
arranjá-los de modo a cobrir um quadrado de lado 1.

Obs: É permitido sobrepor quadrados e parte deles pode ultrapassar os
limites do quadrado a ser coberto.

Solução: Se há um quadrado de área =1, terminou. Se não há, divida os
quadrados em tipos. Tipo n: quadrados com área entre1/4^n e 1/4^(n-1). Então
é fácil ver que precisamos considerar que há no máximo 3 quadrados de cada
tipo, pois se há 4 do tipo n, junte-os e terá um de um tipo n-1. Suponha
então que não temos mais de 4 quadrados do tipo n, para todo n.
Então a soma das áreas é  3 + 3/4 + 3/16 + = 4 (pois há um número
finito de quadrados), uma contradição, pois temos soma =4.  Logo existem 4
quadrados do tipo k, para algum k (considere o menor k com essa
propriedade). Então junte-os e temos um quadrado extra do tipo k-1. Agora, é
só repetir o argumento : certamente há 4 de algum tipo, chame de j esse
tipo. Claramente j  k, junte os quadrados do tipo j e temos mais um do tipo
j-1. Prosseguindo desta forma, temos 4 quadrados do tipo 1, juntando-os,
temos um quadrado de área 1, que cobre o quadrado dado.


Abraços,
 Villard


-Mensagem original-
De: [EMAIL PROTECTED] [EMAIL PROTECTED]
Para: [EMAIL PROTECTED] [EMAIL PROTECTED]
Data: Domingo, 15 de Dezembro de 2002 17:09
Assunto: [obm-l] NOVO MEMBRO E UMA DÚVIDA


Olá, pessoal. Meu nome é Helder Oliveira de Castro e sou um novo membro da
lista. A minha dúvida é sobre o problema No. 5 da OBM 2002 - será que
alguém
pode me ajudar?

_
Voce quer um iGMail protegido contra vírus e spams?
Clique aqui: http://www.igmailseguro.ig.com.br

=
Instruções para entrar na lista, sair da lista e usar a lista em
http://www.mat.puc-rio.br/~nicolau/olimp/obm-l.html
O administrador desta lista é [EMAIL PROTECTED]
=


=
Instruções para entrar na lista, sair da lista e usar a lista em
http://www.mat.puc-rio.br/~nicolau/olimp/obm-l.html
O administrador desta lista é [EMAIL PROTECTED]
=



Re: [obm-l] TFA

2002-12-06 Por tôpico Rodrigo Villard Milet



Procure nos arquivos da lista. O teorema fundamental da álgebra foi 
recentemente discutido aqui.

  -Mensagem original-De: 
  Wagner [EMAIL PROTECTED]Para: [EMAIL PROTECTED] [EMAIL PROTECTED]Data: 
  Sexta-feira, 6 de Dezembro de 2002 10:37Assunto: [obm-l] 
  TFA
  Oi para todos !
  
  
  Alguém sabe de um site onde posso conseguir a 
  prova do Teorema Fundamental da Álgebra?
  
  André T.


Re: [obm-l] Sobre o Teorema Fundamental da Algebra(ajuda)

2002-11-23 Por tôpico Rodrigo Villard Milet



Então... essa é a contradição... vc supõe q P, um polinômio ñ 
constante,ñ tem raízes.. e chega em q ele constante... absurdo, logo ele 
deve ter uma raiz. 

Usando o teorema de Green é bastante legal. Vou colocar a idéia só... e aí 
vc formaliza. Se não conseguir, eu coloco. Considere o polinômio P como uma 
função de C em C. Então, se P(z)= a(n)*z^n +...+a(1)*z+a(0), considere que a(0) 
é diferente de zero, senão é trivial. Temos P(0)=a(0). Agora a idéia é vc 
estudar as imagens de círculos centrados na origem por P. Seja C(r) tal circ com 
raio r. Para r suficientemente pequeno, P(C(r)) não dá nenhuma volta no zero. 
Agora, sabemos que quando r é suficientemente grande, | P(C(r)) | é grande, 
portanto dá pelo menos uma volta no zero (pois |P(z)| vai pra inf qd |z| vai pra 
inf). Então, por continuidade, em algum R temos P(C(R)) passando por zero, o q 
nos dá uma raiz.
Essa é a idéia. Agora, para formalizá-la, vc pode usar o teorema de green, 
junto com a forma de medida de ângulo.
Abraços, 
Villard

  -Mensagem original-De: 
  Johann Peter Gustav Lejeune Dirichlet [EMAIL PROTECTED]Para: 
  [EMAIL PROTECTED] [EMAIL PROTECTED]Data: 
  Sexta-feira, 22 de Novembro de 2002 15:04Assunto: Re: [obm-l] 
  Sobre o Teorema Fundamental da Algebra(ajuda)
  Ola Villard!!Voce tem a demonstraçao por Green?Poe ela aqui pra todo 
  mundo ver!Falando nisso,foi bom voce ter me lembrado deste Teorema de 
  Liouville.Vou pegar a demonstraçao agora(esta esta no livro Variaveis 
  Complexas,de Murray Ralph Spiegel,traduzido por Jose Raimundo Coelho,Coleçao 
  Schaum,Ed.McGraw-Hill): 
  TEOREMA DE LIOUVILLE:se para qualquer ponto z complexo,sabe-se que a funçao 
  f(z) e analitica e limitada(ou seja,existeM real tal que 
  |f(z)|M),entao f(z) deve ser constante. 
  Suponha que o polinomio de grau n0 nao tenha 
  raizes.Entaof(z)=1/Polinomio seriaanalitica,e limitada(f tende a 0 
  quando |z| cresce).Logo e constante,por Liouville.Mas desde quando polinomio e 
  constante? 
  Rodrigo Villard Milet 
  [EMAIL PROTECTED] wrote: 
  

A demonstração mais simples que tem é usando o teorema de Liouville 
(acho q é assim q se escreve)... no entanto conheço uma que usa o teorema de 
Green tb... é mais legal, é claro :)

Abraços, 
Villard

  -Mensagem original-De: 
  Johann Peter Gustav Lejeune Dirichlet [EMAIL PROTECTED]Para: 
  [EMAIL PROTECTED] [EMAIL PROTECTED]Data: 
  Quinta-feira, 21 de Novembro de 2002 14:34Assunto: [obm-l] 
  Sobre o Teorema Fundamental da Algebra(ajuda)
  Nao me lembro mais quem me perguntou sobre isso,mas acho que ja esta na 
  hora de responder.E sobre a existencia de soluçoes complexas de polinomios 
  em C[z]
  Para demonstrar o TFA,vou enunciar esses dois teoremas,que podem ser 
  demonstrados com a ajuda das formulas integrais de Cauchy.Depois eu falo 
  disso em outros e-mails.
  TEOREMA DE ROUCHE:se em uma curva fechada C e sobre ela as funçoes f(z) 
  e g(z)sao analiticas,e |g(z)||f(z)| em C,temos que as funçoes g(z) e 
  f(z)+g(z) tem o mesmo numero de zeros em C(contando multiplicidades).
  Agora considere as funçoes f(z)=polinomio de grau n-1,g(z)=z^n,e 
  considere a superficie C como sendo um circulo centrado na origem de raio 
  R suficientemente grande(maior que 1),de modo que |f(z)|/|g(z)|1 em 
  C.Para encontrar esse raio R,use o fato de que em C nenhum complexo tem 
  comprimento maior que o raio.
  Depois de demonstrar isso,basta ver que g(z) tem n zeros em 
  C
  TRANSIRE SVVM PECTVS MVNDOQUE POTIRE
  CONGREGATI EX TOTO ORBE MATHEMATICI OB SCRIPTA INSIGNIA TRIBVERE
  Fields Medal(John Charles Fields)
  
  
  Yahoo! 
  GeoCitiesTudo para criar o seu site: ferramentas fáceis de 
  usar, espaço de sobra e acessórios.
  TRANSIRE SVVM PECTVS MVNDOQUE POTIRE
  CONGREGATI EX TOTO ORBE MATHEMATICI OB SCRIPTA INSIGNIA TRIBVERE
  Fields Medal(John Charles Fields)
  
  
  Yahoo! GeoCitiesTudo 
  para criar o seu site: ferramentas fáceis de usar, espaço de sobra e 
  acessórios.


Re: [obm-l] Matriz Inversa

2002-11-22 Por tôpico Rodrigo Villard Milet
Se vc sabe um poko de álgebra linear, é fácil...
Olhe A e X como transformações lineares de R^N em R^N. Então X é injetora,
pois dados u,v em R^N, Xu=Xv implica AXu=AXv, logo u=v. Pelo teorema do
núcleo e da imagem, X é sobrejetora, logo é bijetora e portanto possui
inversa. Então existe a transformação X^(-1), que possui matriz X^(-1). Daí
segue trivialmente que A é a inversa de X.
Se vc ñ entendeu essas coisas (é pq ainda ñ viu, é claro), procure o livro
de álgebra linear do Elon, q é de fácil acesso.
Abraços.
 Villard
De: Daniel [EMAIL PROTECTED]
Para: [EMAIL PROTECTED] [EMAIL PROTECTED]
Data: Sexta-feira, 22 de Novembro de 2002 22:21
Assunto: [obm-l] Matriz Inversa


Olá à todos os membros da lista!

Uma pergunta teórica sobre matrizes:

Sejam A e X matrizes quadradas de ordem n e I a
matriz identidade de mesma ordem. Para a equação:
AX = I, posso afirmar que X é a inversa de A, ou
é preciso definir que
AX = XA = I

Grato

Daniel O . Costa

=
Instruções para entrar na lista, sair da lista e usar a lista em
http://www.mat.puc-rio.br/~nicolau/olimp/obm-l.html
O administrador desta lista é [EMAIL PROTECTED]
=


=
Instruções para entrar na lista, sair da lista e usar a lista em
http://www.mat.puc-rio.br/~nicolau/olimp/obm-l.html
O administrador desta lista é [EMAIL PROTECTED]
=



Re: [obm-l] Sobre o Teorema Fundamental da Algebra(ajuda)

2002-11-21 Por tôpico Rodrigo Villard Milet



A demonstração mais simples que tem é usando o teorema de Liouville (acho q 
é assim q se escreve)... no entanto conheço uma que usa o teorema de Green tb... 
é mais legal, é claro :)

Abraços, 
Villard

  -Mensagem original-De: 
  Johann Peter Gustav Lejeune Dirichlet [EMAIL PROTECTED]Para: 
  [EMAIL PROTECTED] [EMAIL PROTECTED]Data: 
  Quinta-feira, 21 de Novembro de 2002 14:34Assunto: [obm-l] 
  Sobre o Teorema Fundamental da Algebra(ajuda)
  Nao me lembro mais quem me perguntou sobre isso,mas acho que ja esta na 
  hora de responder.E sobre a existencia de soluçoes complexas de polinomios em 
  C[z]
  Para demonstrar o TFA,vou enunciar esses dois teoremas,que podem ser 
  demonstrados com a ajuda das formulas integrais de Cauchy.Depois eu falo disso 
  em outros e-mails.
  TEOREMA DE ROUCHE:se em uma curva fechada C e sobre ela as funçoes f(z) e 
  g(z)sao analiticas,e |g(z)||f(z)| em C,temos que as funçoes g(z) e 
  f(z)+g(z) tem o mesmo numero de zeros em C(contando multiplicidades).
  Agora considere as funçoes f(z)=polinomio de grau n-1,g(z)=z^n,e considere 
  a superficie C como sendo um circulo centrado na origem de raio R 
  suficientemente grande(maior que 1),de modo que |f(z)|/|g(z)|1 em C.Para 
  encontrar esse raio R,use o fato de que em C nenhum complexo tem comprimento 
  maior que o raio.
  Depois de demonstrar isso,basta ver que g(z) tem n zeros em C
  TRANSIRE SVVM PECTVS MVNDOQUE POTIRE
  CONGREGATI EX TOTO ORBE MATHEMATICI OB SCRIPTA INSIGNIA TRIBVERE
  Fields Medal(John Charles Fields)
  
  
  Yahoo! GeoCitiesTudo 
  para criar o seu site: ferramentas fáceis de usar, espaço de sobra e 
  acessórios.


[obm-l] Re: [obm-l] questão 4 - IME 2003

2002-11-10 Por tôpico Rodrigo Villard Milet



Vá em www.pensi.com.br . Lá você vai 
encontrar os gabaritos das outras provas tb.
Abraços, Villard

  -Mensagem original-De: 
  Wander Junior [EMAIL PROTECTED]Para: 
  [EMAIL PROTECTED] [EMAIL PROTECTED]Data: 
  Domingo, 10 de Novembro de 2002 11:53Assunto: [obm-l] questão 4 
  - IME 2003
  Resolva a equação:
  
  tg(a) + tg(2a) = 2 . tg(3a) , 
  sabendo-se que a pertence a [0,pi/2).
  
  Obrigado.
  Wander.
  


[obm-l] Re: [obm-l] Recorrência

2002-11-09 Por tôpico Rodrigo Villard Milet
Faça b_{n} = x_{n} - x_{n-1}. A equação dada é equivalente a b_{n} =
n*b_{n-1}.
Logo b_{n} = n! *b_{1} = n! * (x_{1} - x_{0}).
Agora vc tem  x_{n} - x_{n-1} =  n! * (x_{1} - x_{0}). Então basta fazer
somatório de 1 até k dos dois lados que vc tem a fórmula pro x_{n} :
x_{n} = x_{0} + (x_{1} - x_{0})* (1!+2!++n!)
Abraços, Villard
-Mensagem original-
De: Marcelo Souza [EMAIL PROTECTED]
Para: [EMAIL PROTECTED] [EMAIL PROTECTED]
Data: Sábado, 9 de Novembro de 2002 11:33
Assunto: [obm-l] Recorrência


Oi pessoal, como resolvo a recorrência

x_{n}=(n+1)x_{n-1}-nx_{n-2}?

me enrolei pq os coeficientes não são contantes...
falow
[]'s
Marcelo

_
STOP MORE SPAM with the new MSN 8 and get 2 months FREE*
http://join.msn.com/?page=features/junkmail

=
Instruções para entrar na lista, sair da lista e usar a lista em
http://www.mat.puc-rio.br/~nicolau/olimp/obm-l.html
O administrador desta lista é [EMAIL PROTECTED]
=


=
Instruções para entrar na lista, sair da lista e usar a lista em
http://www.mat.puc-rio.br/~nicolau/olimp/obm-l.html
O administrador desta lista é [EMAIL PROTECTED]
=



[obm-l] Questão boba e legal

2002-10-24 Por tôpico Rodrigo Villard Milet




Dêem uma olhada nessa questão que eu inventei (sem 
querer)... é bastante fácil, mas achei o resultado um tanto curioso :
É dada uma caixa em forma de prisma reto de base 
retangular de dimensões a e b. Apenas uma das arestas da base está presa no chão 
(uma de medida a), enquanto as outras estão apenas apoiadas. Sejah a face da caixa que é paralela à base e não está no chão. 
Um indivíduo empurra a caixa, fazendo-a então cair no chão. Determine o volume 
da região varrida por h durante a queda em função de a 
e b.
Abraços, 
Villard


[obm-l] Questão boba e legal

2002-10-24 Por tôpico Rodrigo Villard Milet
=
Instruções para entrar na lista, sair da lista e usar a lista em
http://www.mat.puc-rio.br/~nicolau/olimp/obm-l.html
O administrador desta lista é [EMAIL PROTECTED]
=



[obm-l] Re: [obm-l] dúvida

2002-09-28 Por tôpico Rodrigo Villard Milet



Escreva 610 na base 2 : 610 = (1001100010)_2. Como sabemos que a 
representação na base 2 é única, ele acertou as perguntas 2,6,7 e 10.
Villard

  -Mensagem original-De: 
  Mário Pereira [EMAIL PROTECTED]Para: 
  [EMAIL PROTECTED] [EMAIL PROTECTED]Data: 
  Sábado, 28 de Setembro de 2002 11:22Assunto: [obm-l] 
  dúvida
  
  
  Olá, se alguém puder, me dê uma 
dica:
  
  Em um jogo de televisão, um candidato deve 
  responder a 10 perguntas. A primeira vale 1 ponto, a segunda vale 2 pontos, e 
  assim, sucessivamente, dobrando sempre. O candidato responde a todas as 
  perguntas e ganha os pontos correspondentes às respostas que acertou, mesmo 
  que erre algumas. Se o candidato obteve 610 pontos, quantas perguntas 
  acertou?
  
  Obrigado, 
  
  Mário. 


[obm-l] Re: [obm-l] 3 problemas olímpicos

2002-09-06 Por tôpico Rodrigo Villard Milet

Eu enviei a solução do 3 pra eureka 12. Dê uma olhada em www.obm.org.br .

-Mensagem original-
De: fredericogomes [EMAIL PROTECTED]
Para: [EMAIL PROTECTED] [EMAIL PROTECTED]
Data: Sexta-feira, 6 de Setembro de 2002 02:21
Assunto: [obm-l] 3 problemas olímpicos


1-(Ucrânia 1992)- Demonstrar que não existem soluções
reais do sistema:
  { x^2 + 4yz + 2z=0
  { x   + 2xy + 2z^2  =0
  { 2xz + y^2 + y + 1 =0


2-(China 1993) Achar todas as ternas (x,y,z) de inteiros
não negativos tais que: 7^x + 1 = 3^y + 5^z.

obs: é óbvio que (0,0,0) e (1,1,1) são soluções e que
não temos mais nenhuma solução que envolva inteiro(s)
nulo(s), neste caso podemos admitir x,y,z =1

3-(Iran 1993) Encontrar todos os primos ímpares p tais
que  [ 2^(p-1) - 1 ] / p é  um quadrado perfeito

Ficarei imensamente grato se tiver pelo menos um destes
três resolvidos.

  []´s Frederico.


__
AcessoBOL, só R$ 9,90! O menor preço do mercado!
Assine já! http://www.bol.com.br/acessobol


=
Instruções para entrar na lista, sair da lista e usar a lista em
http://www.mat.puc-rio.br/~nicolau/olimp/obm-l.html
O administrador desta lista é [EMAIL PROTECTED]
=



[obm-l] Re: [obm-l] Re: [obm-l] Re: [obm-l] Re: Área do triângulo

2002-08-25 Por tôpico Rodrigo Villard Milet

Não é difícil... apenas parece...
Dado um triângulo ABC, com medianas AM, BN, CL e baricentro G, prolongue AM
até P de modo que GM=MP. Então é fácil ver que o triângulo GPC tem lados
iguais a 2/3 das medianas de ABC ( Verifique ! ). Como a área de GMC é S/6,
a área de GPC têm área S/3. Daí segue que a área procurada é
(9/4)*(S/3)=(3/4)S

Agora é bem fácil pensar na construção com régua e compasso, olhando para a
construção feita acima.

Abraços
Villard
-Mensagem original-
De: Vinicius José Fortuna [EMAIL PROTECTED]
Para: [EMAIL PROTECTED] [EMAIL PROTECTED]
Data: Sábado, 24 de Agosto de 2002 23:30
Assunto: [obm-l] Re: [obm-l] Re: [obm-l] Re: Área do triângulo


Pô, coitado do Renato. Com o atraso que o gerenciador da lista tem para
enviar os e-mails acabou tendo um monte de gente corrigindo ele.

Bruno, com relação ao teorema que vc citou, ele tem algum nome especial
para
que eu posso buscá-lo em outras fontes?

Uma outra pergunta. Dada as medidas das medianas, é possível construir o
triângulo com régua e compasso? Como?

Obrigado

Vinicius Fortuna

- Original Message -
From: Bruno F. C. Leite [EMAIL PROTECTED]
To: [EMAIL PROTECTED]
Sent: Saturday, August 24, 2002 9:12 PM
Subject: Re: [obm-l] Re: [obm-l] Re: Área do triângulo


 Oi,

 Posso estar falando uma besteira feia, mas quando eu estudava geometria
 plana (há 3 anos) eu acho que tinha um teorema que dizia que dado um
 triangulo, podemos montar um triângulo com suas medianas e a razão entre
as
 áreas destes triangulos é 3/4.

 Se isto for verdade, o problema fica fácil.

 Bruno Leite
 http://www.ime.usp.br/~brleite

 At 20:04 24/08/02 -0300, you wrote:
 Renato,
 x, y e z são as medianas do triângulo e não seus lados!
 Um abraço!
 Eduardo.
 
 From: Renato Lira [EMAIL PROTECTED]
   Para saber se o triangulo realmente existe, tem que obedecer as
seguintes
   regras: x + y  z ; x + z  y ; y + z  x
  
   Para saber sua área sabendo somente os lados: seja p o semi perimetro
   (x+y+z)/2
  
   S = sqrt[p(p-x)(p-z)(p-y)]
  
  
  
   - Original Message -
   From: Vinicius José Fortuna [EMAIL PROTECTED]
   To: [EMAIL PROTECTED]
   Sent: Saturday, August 24, 2002 7:36 PM
   Subject: [obm-l] Área do triângulo
  
  
Uma das questões do último campeonato de programação do site de
 Valladolid
(http://acm.uva.es/problemset) era o seguinte:
   
Dados os tamanhos x, y, z das medianas de um triângulo, calcular
sua
 área
   ou
dizer que tal triângulo não existe.
   
Alguém tem alguma idéia de como resolver?
   
Obrigado
   
Vinicius Fortuna
IC-Unicamp


=
Instruções para entrar na lista, sair da lista e usar a lista em
http://www.mat.puc-rio.br/~nicolau/olimp/obm-l.html
O administrador desta lista é [EMAIL PROTECTED]
=


=
Instruções para entrar na lista, sair da lista e usar a lista em
http://www.mat.puc-rio.br/~nicolau/olimp/obm-l.html
O administrador desta lista é [EMAIL PROTECTED]
=



[obm-l] Re: [obm-l] questão-funções

2002-08-15 Por tôpico Rodrigo Villard Milet




Considere H(x) = [f(x)]^2+[g(x)]^2. Ento H`(x) 
= 2f(x)*f`(x) + 2g(x)*g`(x) = 0, pois f'(x)=g(x), g'(x)= -f(x). Ento, 
temos que H(x)  uma constante, logo H(x) = H(0), para todo 
x.
[f(x)]^2+[g(x)]^2 = 
[f(0)]^2 + [g(0)]^2 = 1, se f(0)=0 e g(0)=1.
Voc se 
equivocou quando disse g(0)=0.
Abraos, Villard

-Mensagem original-De: 
[EMAIL PROTECTED] [EMAIL PROTECTED]Para: 
[EMAIL PROTECTED] [EMAIL PROTECTED]Data: 
Quinta-feira, 15 de Agosto de 2002 10:47Assunto: [obm-l] 
questo-funes 
Ol pessoal da lista,algum poderia me dar uma ajuda na 
questo abaixo? 
 Dada 
duas funes f e g cujas derivadas f' e 
g' satisfazem as equaesf'(x)=g(x), g'(x)= -f(x), 
f(0)=0 e g(0)=0 , para todo x pertencente a algum intevalo aberto 
j contendo 0.Por exemplo, as equaes so 
satisfeitas quando f(x)=senx e 
g(x)=cosx. 
Prove que [f(x)]^2 + [g(x)]^2 = 1 para todo x pertencente 
a 
j. 
Agradeo desde j qualquer ajuda,um 
abrao, 
Bruno Moss.


Re: [obm-l] Discussao dos problemas da IMO

2002-07-30 Por tôpico Rodrigo Villard Milet

Essa idéia de fazer em dois dias é boa, pois cada um tem sua disponibilidade
de horários... eu só posso na sexta...
Abraços,
 Villard
-Mensagem original-
De: Carlos Gustavo Tamm de Araujo Moreira [EMAIL PROTECTED]
Para: [EMAIL PROTECTED] [EMAIL PROTECTED]
Data: Terça-feira, 30 de Julho de 2002 15:41
Assunto: Re: [obm-l] Discussao dos problemas da IMO


Eu tinha proposto na sexta por sugestao do Marcio.O Marcelo estava no
IMPA e disse que tambem preferia sexta.Eu nao tenho nenhum problema na
segunda,entretanto.Talvez seja bom o pessoal do Rio se manifestar sobre que
dia prefere.Por outro lado nao vejo problema em fazer uma reuniao na sexta
e
outra na segunda,e discutir tambem outros problemas,alem dos da IMO,para
aproveitar a animacao do pessoal.O que voces acham ?
Abracos,
Gugu


Eu posso participar se for na segunda-feira. Na sexta é mais difícil.

Luciano.

At 15:12 29/07/02 -0300, you wrote:
Caros colegas,
Por sugestao do Marcio vamos fazer uma reuniao informal na
sexta-feira
(2/8) as 14:00 no IMPA para discutir os problemas da IMO deste ano.Tragam
suas solucoes...
Abracos,
Carlos Gustavo Moreira (Gugu)


=
Instruções para entrar na lista, sair da lista e usar a lista em
http://www.mat.puc-rio.br/~nicolau/olimp/obm-l.html
O administrador desta lista é [EMAIL PROTECTED]
=

=
Instruções para entrar na lista, sair da lista e usar a lista em
http://www.mat.puc-rio.br/~nicolau/olimp/obm-l.html
O administrador desta lista é [EMAIL PROTECTED]
=

=
Instruções para entrar na lista, sair da lista e usar a lista em
http://www.mat.puc-rio.br/~nicolau/olimp/obm-l.html
O administrador desta lista é [EMAIL PROTECTED]
=


=
Instruções para entrar na lista, sair da lista e usar a lista em
http://www.mat.puc-rio.br/~nicolau/olimp/obm-l.html
O administrador desta lista é [EMAIL PROTECTED]
=



[obm-l] IMO, dia 2, Q5 (solução)

2002-07-27 Por tôpico Rodrigo Villard Milet




Segue a minha soluo para 
a quinta questo dessa IMO. Confiram :),( se algum tiver 
pacincia ). (f(x)+f(z))*(f(y)+f(t)) = f(xy-zt) + f(xt+yz)
Primeiramente faa x=z=0 : 2f(0) * ( f(y) + 
f(t) ) = 2f(0), logo ou f(0)=0, ou f(y)+f(t) = 1, para todos y,t reais e em 
particular quando y=t, temos f(y)=1/2, para todo y real, o que  uma 
soluo particular. Assuma ento f(0)=0.
Faa z=t=0 : f(xy)=f(x)*f(y). Ento, 
fazendo y=1, f(x)=f(x)*f(1), logo temos f(1)=1 ou f(x)=0, para todo x, o que 
 outra soluo particular. Faa y=t=1 e x=0 : 2f(z) = f(-z)+f(z), logo f  par. 
Ento, precisamos nos preocupar apenas com a parte positiva.
Na equao inicial, temos : 
f(xy)+f(xt)+f(yz)+f(zt) = f(xy-zt) + f(xt+yz), ento 
f(a)+f(b)+f(c)+f(d)=f(a-d)+f(b+c), desde que ad=bc. Colocando a=c=mx, b=d=x , 
temos 2f(mx)+2f(x)=f((m-1)x) + f((m+1)x), o que prova por induo 
que f(mx)=m^2 * f(x), para todo m inteiro. Logo f(m)=m^2, para todo m inteiro. 
f(p/q)=f(p)/f(q), pois  multiplicativa, logo f(p/q)=(p/q)^2, 
ento f(m)=m^2, para todo m racional. Vamos mostrar que f  
montona crescente em R+. Faa y=x,t=z : (f(x)+f(z))^2 = 
f(x^2-z^2)+f(2xy). Faa y=z,t=x : (f(x)+f(z))^2=f(x^2+z^2). Juntando as 
duas ltimas, temos que f(a)=f(b)+f(c), quando a^2 = b^2+c^2, logo f 
 crescente (pois se ab0, ento existe c0 tal que 
a^2=b^2+c^2, logo f(a)=f(b)+f(c) f(b), pois f(c)=f(sqrt(c))^2=0. Em 
particular f  positiva.).
Como f  crescente em R+ e f(m)=m^2 nos 
racionais, ento  fcil mostrar que f(x)=x^2 para todos x 
em R+, logo para todos x em R, pois f  par. Sejam a(n) e b(n) duas sequncias de racionais convergindo para um 
irracional x0, tais que 0a(n) x b(n). Ento como f 
 montona crescente, a(n)^2 = f(x) = b(n)^2 e fazendo n 
tender a infinito, temos a(n) e b(n) tendendo a x, logo f(x)=x^2 tambm 
nos irracionais.
Resposta : f(x)==0 ; f(x)==1/2 ; f(x)==x^2 
.

Abraos, 
Villard


[obm-l] IMO!?!?

2002-07-24 Por tôpico Rodrigo Villard Milet




Onde eu acho a prova da imo de hj ?!? Se 
algum j tiver, por favor mande para a lista.
Obrigado !

Villard


[obm-l] Re: [obm-l] Teoria dos números....

2002-07-22 Por tôpico Rodrigo Villard Milet




1) Olhe mod6. Se p  primo ento ou p=3 
ou p==+-1mod6. Neste ltimo caso, temos p^2+8==3mod6, logo p^2+8  
mltiplo de 3 nesse caso. Ento s resta p=3, logo p^2+8=17 
q  primo. e p^3+4=31 q  primo. Acabou. Aqui  
fcil ver que voc deveria primeiro achar todos os p tais que p e 
p^2+8 so primos, pois seno teramos uma mquina 
para gerar primos muito simples.

2) a=pu e b=pv, com mdc(u,v)=1.Ento E = mdc(a^3,b) = 
mdc(p^3*u^3,pv). Agora  fcil... Seja j o expoente de p na 
fatorao prima de v. 
Se j=0, E=p., pois como mdc(u,v)=1, temos mdc(v,p^3*u^3)=1
Se j=1, E=p^2, pois como mdc(u,v)=1, temos mdc(v,p^3*u^3)=p
Se j=2, E=p^3, pois como mdc(u,v)=1, temos mdc(v,p^3*u^3)=p^2
Se j2, E=p^3, pois pv tem ao menos 4 fatores p e nenhum fator comum com 
u^3.

Abraos, 
Villard


-Mensagem original-De: 
[EMAIL PROTECTED] [EMAIL PROTECTED]Para: [EMAIL PROTECTED] [EMAIL PROTECTED]Data: 
Segunda-feira, 22 de Julho de 2002 14:36Assunto: [obm-l] 
Teoria dos nmerosE 
a rapaziada...estou mandando alguns problemas que parecem simples, 
mas me perco na hora de colocar no papel.1) Se p e p^2+8 so 
ambos nmeros primos, prove que p^3+4 tambm .2) 
se mdc(a,b )=p, onde p  primo, mostre que mdc(a^3,b)=p, p^2 ou 
p^3. 
obrigado, 
Korshinoi 


Re: [obm-l] Geo Plana..

2002-07-04 Por tôpico Rodrigo Villard Milet




O ngulo BMC  60. Ento construa 
um tringulo equiltero PMQ, com P mdio de MB e Q em MC. 
Temos AM=MP=PB=MQ. Olhe para o tringulo BMQ. Nele, a mediana relativa a 
MB  igual a metade de MB, logo o ngulo MQB  reto. O mesmo 
vale para o ngulo AQP. Ento MBQ=30 e MAQ=30, logo QAC=15, 
ento o tringulo ACQ  issceles, da AQ=CQ. 
Mas tambm sabemos que BQ=AQ, pois os tringulos AQP e BMQ 
so congruentes (j que tm os mesmos ngulos e 
tm hipotenusas iguais). Logo BQ=CQ... BCQ  issceles 
implica :
a-30=180-(60+a) ... a = 75.
Abraos, Villard.
PS: CM NO trissecta o ngulo C.

-Mensagem original-De: 
Igor Castro [EMAIL PROTECTED]Para: 
obm-lista [EMAIL PROTECTED]Data: 
Quarta-feira, 3 de Julho de 2002 19:43Assunto: [obm-l] Geo 
Plana..
Ol amigos.. alguem pode dar uma ajuda 
nesse problema de geometria que no est saindo? 


A medida do angulo a na figura, 
sendo AM a metade de MB, : (segue figura em 
anexo)


[obm-l] Questão : série/sequência

2002-06-08 Por tôpico Rodrigo Villard Milet




Talvez a questo que estou 
enviando seja fcil... mas quero ver se algum d alguma 
soluo elegante pra ela... l vai :
Sabe-se que somatrio { a(n) } 
converge. Calcular lim [(1/n)*somatrio(k*a(k))], onde o somatrio 
vai de 1 at n e o limite  qd n- +oo.
Abraos,
 
Villard


Re: [obm-l] Desafio

2002-06-07 Por tôpico Rodrigo Villard Milet

Use que 1+a(i) =2sqrt[a(i)]. Fazendo o produto dessas n equações, temos que
P =2^n * sqrt[ produto a(i) ] = 2^n * 2 = 2^(n+1). RESPOSTA : C.
Villard
-Mensagem original-
De: Eduardo Casagrande Stabel [EMAIL PROTECTED]
Para: [EMAIL PROTECTED] [EMAIL PROTECTED]
Data: Quinta-feira, 6 de Junho de 2002 23:09
Assunto: Re: [obm-l] Desafio


Caro Bruno,

a notação que você usou não está muito legível. Seria melhor adotar índices
para os a's, por exemplo: a_1, a_2, a_3, ..., a_n. Para fazer exponenciação
geralmente se usa ^, aí as alternativas seriam P2^(n+3), P5^n, e assim
por diante.

Quanto ao problema. Existe uma desigualdade, que aprendi a demonstrar por
indução (e talvez você já conheça ou queira provar como exercício) que diz
que se a_1, a_2, ..., a_n são números não-negativos então

(1 + a_1)*(1 + a_2)*...*(1 + a_n) = 1 + (a_1*a_2*...*a_n)

com a igualdade se e so se todos os a_i's forem iguais a zero.

No caso do seu problema. Temos

P = (1 + a_1)*(1 + a_2)*...*(1 + a_n)  1 + (a_1*a_2*...*a_n) = 5.

Isso claramente não resolve o problema. Uma estratégia mais interessante me
parece procurar pelo valor mínimo de P, após fixado o n.

Fazendo a multiplicação, temos

P = (1 + a_1)*(1 + a_2)*...*(1 + a_n) = 1 + [a_1+a_2+...+a_n] +
[a_1a_2+a_1a_3+...+a_(n-1)a_n] + ... + [a_1a_2...a_n]

No primeiro colchetes temos os n termos a's solitários.
No segundo colchetes temos os produtos de pares de a's.
No terceiro, o produto de trincas. E assim por diante.
Vamos aplicar a desigualdade: média aritmética = média geométrica em cada
um dos colchetes.

P = 1 + n*[RAIZ_n {a_1a_2...a_n}] + n(n-1)/2*[RAIZ_n(n-1)/2
{(a_1a_2...a_n)^(n-1)}] + ... + [a_1a_2...a_n]

De forma mais compacta

P = 1 + SOMATÓRIO{ k=1...n   :C(n,k) * RAIZ_C(n,k) {
(a_1a_2...a_n)^(C(n-1,k-1))  } } =
1 + SOMATÓRIO{ k=1...n   :C(n,k) * (RAIZ_n (4^k) }
= (1 + RAIZ_n(4))^n

((Revisem as contas, fiz de modo simplificado))

Basta mostrar que a igualdade ocorre se e somente se a_1=a_2=...a_n, mas
isso é claro por termos usado a desigualdade média aritmética e geométrica.

Portanto P = (1 + RAIZ_n(4))^n e a igualdade pode ocorrer para cada n.

Com isso fica fácil de ver que qualquer exponencial do tipo a^n (onde a1)
vai acabar superando P para algum n suficientemente grande, repare que 1 +
RAIZ_n(4), a base da nossa exponencial se aproxima de 1 a medida que n
cresce. De modo que nem a) nem b) nem c) nem d) são verdadeiras. Logo a
alternativa correta é e).

Um abraço!

Eduardo Casagrande Stabel. Porto Alegre, RS.


From: Bruno

Eu não consegui fazer este exercício do ITA e desafio todos dessa lista:
Suponha a', a'', ., an  são números reais positivos, com n2 e que
a'.a''.a'''an=4
Nesta situação, a repeito do produto:
P=(1+a')(1+a'')...(1+an)  temos:
  n+3
a.)P2
   n
b.)P5
 n+1
   c.)P2
 n+1
d.)P5
e.)n.d.a.



=
Instruções para entrar na lista, sair da lista e usar a lista em
http://www.mat.puc-rio.br/~nicolau/olimp/obm-l.html
O administrador desta lista é [EMAIL PROTECTED]
=


=
Instruções para entrar na lista, sair da lista e usar a lista em
http://www.mat.puc-rio.br/~nicolau/olimp/obm-l.html
O administrador desta lista é [EMAIL PROTECTED]
=



[obm-l] Re: [obm-l] diferença de raízes

2002-06-07 Por tôpico Rodrigo Villard Milet

Só complementando a msg anterior... a resposta então é 3*sqrt(5).
-Mensagem original-
De: Rafael WC [EMAIL PROTECTED]
Para: [EMAIL PROTECTED] [EMAIL PROTECTED]
Data: Sexta-feira, 7 de Junho de 2002 07:19
Assunto: [obm-l] diferença de raízes


Olá Pessoal!

Já tem um mês que eu tento resolver esse exercício sem
sucesso. Se alguém conseguir algum avanço, por favor
escreva!

Qual a diferença entre a maior e a menor raiz da
equação:
x^2 + (9x²)/(x+3)² = 27

Obrigado.

Rafael.


=
Rafael Werneck Cinoto
   ICQ# 107011599
 [EMAIL PROTECTED]
   [EMAIL PROTECTED]
   [EMAIL PROTECTED]
http://www.rwcinoto.hpg.com.br/

__
Do You Yahoo!?
Yahoo! - Official partner of 2002 FIFA World Cup
http://fifaworldcup.yahoo.com
=
Instruções para entrar na lista, sair da lista e usar a lista em
http://www.mat.puc-rio.br/~nicolau/olimp/obm-l.html
O administrador desta lista é [EMAIL PROTECTED]
=


=
Instruções para entrar na lista, sair da lista e usar a lista em
http://www.mat.puc-rio.br/~nicolau/olimp/obm-l.html
O administrador desta lista é [EMAIL PROTECTED]
=



[obm-l] Re: [obm-l] diferença de raízes

2002-06-07 Por tôpico Rodrigo Villard Milet

A equação dada é equivalente a :
[x - 3x/(x+3)]² = 27 - 6x²/(x+3), ou seja, [x²/(x+3)]² = 27 - 6x²/(x+3).
Agora faça x²/(x+3) = y. Temos que y² + 6y - 27 = 0 e segue que y = -9 ou y
= 3.
(i) x²/(x+3) = -9 ... x² + 9x + 27 = 0, que ñ dá raízes reais...
(ii) x²/(x+3) = 3 ... x² - 3x - 9 = 0 ... que dá x = [3+-3 sqrt(5)]/2

Talvez tenha alguma conta errada... confira !
Abraços,
  Villard
-Mensagem original-
De: Rafael WC [EMAIL PROTECTED]
Para: [EMAIL PROTECTED] [EMAIL PROTECTED]
Data: Sexta-feira, 7 de Junho de 2002 07:19
Assunto: [obm-l] diferença de raízes


Olá Pessoal!

Já tem um mês que eu tento resolver esse exercício sem
sucesso. Se alguém conseguir algum avanço, por favor
escreva!

Qual a diferença entre a maior e a menor raiz da
equação:
x^2 + (9x²)/(x+3)² = 27

Obrigado.

Rafael.


=
Rafael Werneck Cinoto
   ICQ# 107011599
 [EMAIL PROTECTED]
   [EMAIL PROTECTED]
   [EMAIL PROTECTED]
http://www.rwcinoto.hpg.com.br/

__
Do You Yahoo!?
Yahoo! - Official partner of 2002 FIFA World Cup
http://fifaworldcup.yahoo.com
=
Instruções para entrar na lista, sair da lista e usar a lista em
http://www.mat.puc-rio.br/~nicolau/olimp/obm-l.html
O administrador desta lista é [EMAIL PROTECTED]
=


=
Instruções para entrar na lista, sair da lista e usar a lista em
http://www.mat.puc-rio.br/~nicolau/olimp/obm-l.html
O administrador desta lista é [EMAIL PROTECTED]
=



[obm-l] Re: [obm-l] Variação na questão do IME: x=sqrt(a-sqrt(a-x))

2002-06-03 Por tôpico Rodrigo Villard Milet

Fazendo y = sqrt(a-x), temos que x = sqrt(a-y).
Daí, segue o sistema :
y² = a - x   (i)
x² = a - y   (ii)
Subtraindo... (y-x)(y+x) = y-x.
Então, ou x=y, ou x+y=1. A pergunta é : quando que x+y=1 é impossível ?
x+y=1 ... y = 1-x ...  1-x = sqrt(a-x) ... 1 - 2x + x² = a - x ... x² - x +
(1-a) = 0. Essa equação não possui raízes reais se e só se 1 - 4(1-a)  0,
ou seja, a  3/4. E é fácil ver que se a0, o problema não tem solução. Logo
0  a  3/4.
Bem... para achar mais a que são solução para o seu problema, eu deveria
aceitar o caso em que a equação x² - x + (1-a) = 0 admite apenas soluções
negativas, pois é claro que x = 0 ( Além disso, x = a ).
Seja f(x) = x² - x + (1-a). A soma das raízes é positiva, logo só podemos
ter uma negativa. Então o caso que nos interessa é qd uma raiz é 0 e outra
a. Assim, temos que f(0)  0 e f(a)  0.
f(0)  0 ... 1-a  0... a  1
f(a)  0 ... a² - 2a + 1...
Então não temos problemas com isso... parece que a solução é de fato 0  a 
3/4.
Aguardo comentários.
Abraços,
  Villard





-Mensagem original-
De: Ralph Teixeira [EMAIL PROTECTED]
Para: Obm (E-mail) [EMAIL PROTECTED]
Data: Segunda-feira, 3 de Junho de 2002 15:09
Assunto: [obm-l] Variação na questão do IME: x=sqrt(a-sqrt(a-x))


 Olá, galera.

 Um colega nosso da lista, o Cláudio, destacou que eu havia me
enganado quando disse que a equação

 x=sqrt(5-sqrt(5-x))

 tem *DUAS* soluções. Ele tem razão -- apesar de eu ainda defender o
fato de que você NÃO PODE SIMPLESMENTE DIZER QUE x=sqrt(5-x), o meu método
acaba por gerar apenas uma raiz de qualquer forma (eu havia cometido um
erro
de álgebra que o Cláudio encontrou).

 Por exemplo, a equação x=sqrt(0.8-sqrt(0.8-x)) tem 3 raízes reais
(você consegue encontrá-las?). Apenas *1* delas é encontrada fazendo
x=sqrt(0.8-x).

 Minha perguntinha para a galera é então: para que valores de a as
equações
x=sqrt(a-sqrt(a-x)) e x=sqrt(a-x) são equivalentes?

 Divirtam-se!

 Abraço,
 Ralph
=
Instruções para entrar na lista, sair da lista e usar a lista em
http://www.mat.puc-rio.br/~nicolau/olimp/obm-l.html
O administrador desta lista é [EMAIL PROTECTED]
=


=
Instruções para entrar na lista, sair da lista e usar a lista em
http://www.mat.puc-rio.br/~nicolau/olimp/obm-l.html
O administrador desta lista é [EMAIL PROTECTED]
=



[obm-l] Caminhos no quadriculado

2002-05-30 Por tôpico Rodrigo Villard Milet




Gostaria de saber se h uma frmula 
fechada para a resposta desse problema em funo de n 
:
Dado um quadriculado n x n, quantos so os 
caminhos que saem do canto inferior esquerdo e chegam ao canto superior direito, 
de forma que o caminho no passe duas vezes pelo mesmo lugar. O caminho 
s pode ser feito sobre os lados dos quadradinhos 1x1 do 
quadriculado.

Abraos, 
 
Villard


[obm-l] Re: [obm-l] fração irredutível

2002-04-30 Por tôpico Rodrigo Villard Milet

Diga pra ele o seguinte :
Uma fração é irredutível se vc já cancelou tudo que era possível ( ou seja,
o q vc disse, que os caras devem ser primos entre si ). Então, se a/b é
irred, então b/a tb o é. Daí, (5n+6)/(n-13) é irred.
Mas (5n+6)/(n-13) = [5(n-13)+71]/(n-13) = 5 + 71/(n-13). É fácil ver agora
que 71/(n-13) deve ser irred. Como você quer o menor inteiro positivo, é
fácil ver que n=1 gera a fração -71/12, que é irred.
Minha resposta é então n=1.
Você pode verificar isso logo de cara. Coloque n=1 na fração dada... vai
dar -12/11, que é irred, e n=1 com certeza é o menor inteiro positivo.


Abraços ,
 Villard
-Mensagem original-
De: Rafael WC [EMAIL PROTECTED]
Para: OBM [EMAIL PROTECTED]
Data: Terça-feira, 30 de Abril de 2002 19:11
Assunto: [obm-l] fração irredutível


Precisaria resolver isso para um aluno do segundo
grau:


o menor inteiro positivo n para o qual
(n - 13)/(5n + 6)
é uma fração não nula e irredutivel é...

Resposta = 84

Eu sei que n - 13 e 5n + 6 devem ser primos entre si,
mas isso não me parece coisa de ensino médio...

Rafael.

=
Rafael Werneck Cinoto
   ICQ# 107011599
 [EMAIL PROTECTED]
   [EMAIL PROTECTED]
   [EMAIL PROTECTED]
http://www.rwcinoto.hpg.com.br/

__
Do You Yahoo!?
Yahoo! Health - your guide to health and wellness
http://health.yahoo.com
=
Instruções para entrar na lista, sair da lista e usar a lista em
http://www.mat.puc-rio.br/~nicolau/olimp/obm-l.html
O administrador desta lista é [EMAIL PROTECTED]
=


=
Instruções para entrar na lista, sair da lista e usar a lista em
http://www.mat.puc-rio.br/~nicolau/olimp/obm-l.html
O administrador desta lista é [EMAIL PROTECTED]
=



[obm-l] Re: [obm-l] duvidas fatoração

2002-04-24 Por tôpico Rodrigo Villard Milet

Acho que não é bem assim...

= (x^2+y^2-z^2)^2 -(2xy)^2 = [ (x+y)^2 -z^2 ] * [ (x-y)^2 - z^2 ]
= -(x+y+z)(x+y-z)(x-y+z)(-x+y+z) ok, até aqui tá igual. Mas vc usou o fato
de x, y, z serem positivos, o que não é dado...
Sejam P=x+y+z, Q= -x+y+z, R=x-y+z e S=x+y-z. Daí, A=-PQRS.
É fácil ver que P==Q==R==S (mod2), logo têm a mesma paridade. Se forem todos
ímpares, A será ímpar, portanto diferente de 2000. Então P, Q, R e S são
pares.
2000 = (2^4)*5^3 ou seja, temos 4 fatores 2 e 3 fatores 5 para distribuir
entre P,Q, R e S. Os fatores 2 já estão distribuídos. Como são 4 caras e 3
cincos, alguém fica sem nenhum 5, ou seja, é igual a +-2.
P+Q+R+S = -2(x+y+z)
As fatorações possíveis são ( a menos da ordem e do sinal de cada fator ) :
2*2*2*250, 2*2*10*50, 2*10*10*10.
Vemos que em todos os casos a soma não é múltipla de 5 ( independente dos
sinais dos fatores ), então x+y+z = +-2.
Se x+y+z = 2, A= -2(2-2z)(2-2y)(2-2x) = 16*(z-1)(y-1)(x-1). Mas agora é
fácil, pois as únicas possibilidades são 16*1*1*125, 16*1*5*25 e 16*5*5*5
 a menos da ordem e dos sinais ). Como (x-1)+(y-1)+(z-1)= -1, temos que a
terceira possibilidade é ímpossível, pois para qq escolha de sinais a soma é
múltipla de 5, logo não é -1. Na segunda possibilidade, temos
obrigatoriamente um fator -1, mas isso quer dizer que um dos x, y, z é zero.
Suponha sem perdas que seja x. Daí, A = y^4+z^4-2(yz)^2 = (y^2-z^2)^2, que é
quadrado, logo não pode ser 2000. E para a primeira possibilidade, é fácil
ver que a soma de 1, 1 e 125 ( podendo mudar o sinal), só pode ser 2,0,-2
mod5, logo não é -1. Então o caso x+y+z = 2 está esgotado.
Se x+y+z = -2, então A = 2(-2-2x)(-2-2y)(-2-2z) = -16(x+1)(y+1)(z+1). É bem
parecido com o caso anterior. As possibilidades são as mesmas que as do caso
anterior, no entanto, devemos ter obrigatoriamente álguem negativo. Assim,
esgotando todos os casos, terminamos a prova.
Ainda espero que tenha uma forma mais rápida de fazer isso, pq há um tempo
eu tinha pensado nesse problema e consegui fazê-lo mais rapidamente... qq
coisa, ou erro, avisem.
Abraços,
 Villard
-Mensagem original-
De: Marcelo Souza [EMAIL PROTECTED]
Para: [EMAIL PROTECTED] [EMAIL PROTECTED]
Data: Terça-feira, 23 de Abril de 2002 10:20
Assunto: Re: [obm-l] duvidas fatoração



1.Fatore a expressão A=x^4 +y^4 +z^4 -2(x^2)(y^2) -2(y^2)(z^2) -
-2(z^2)(x^2) e mostre que a equação A=2000 não possui solução
inteira.
A=(x^2-y^2-z^2)^2 -4y^2z^2
A=(x^2-y^2-z^2-2yz)(x^2-y^2-z^2+2yz)
A=(x^2-(y+z)^2)(x^2-(y-z)^2)
A=(x+y+z)(x-y-z)(x+y-z)(x-y+z)
Por desigualdade triangular, dah pra ver que este numero sempre eh
negativo,
logo A0, para x y z positivos. Acho que eh por isso que naum tem solucao
pra 2000
falow.
marcelo



_
Una-se ao maior serviço de email do mundo: o MSN Hotmail.
http://www.hotmail.com

=
Instruções para entrar na lista, sair da lista e usar a lista em
http://www.mat.puc-rio.br/~nicolau/olimp/obm-l.html
O administrador desta lista é [EMAIL PROTECTED]
=


_
Get your FREE download of MSN Explorer at http://explorer.msn.com/intl.asp.

=
Instruções para entrar na lista, sair da lista e usar a lista em
http://www.mat.puc-rio.br/~nicolau/olimp/obm-l.html
O administrador desta lista é [EMAIL PROTECTED]
=



=
Instruções para entrar na lista, sair da lista e usar a lista em
http://www.mat.puc-rio.br/~nicolau/olimp/obm-l.html
O administrador desta lista é [EMAIL PROTECTED]
=



[obm-l] Re: [obm-l] duvidas fatoração

2002-04-24 Por tôpico Rodrigo Villard Milet

x=5, y=z=2 : A=9*1*5*5 não é negativo então isso não vale.

-Mensagem original-
De: Rafael WC [EMAIL PROTECTED]
Para: [EMAIL PROTECTED] [EMAIL PROTECTED]
Data: Quarta-feira, 24 de Abril de 2002 18:30
Assunto: Re: [obm-l] duvidas fatoração


Sei que essa resposta já foi contestada, mas eu
gostaria de saber como é que o Marcelo usou
desigualdade triangular para x, y, z positivos. Mesmo
assim não consigo ver que
A=(x+y+z)(x-y-z)(x+y-z)(x-y+z) é negativo.

Rafael.

--- Marcelo Souza [EMAIL PROTECTED] wrote:

 1.Fatore a expressão A=x^4 +y^4 +z^4 -2(x^2)(y^2)
 -2(y^2)(z^2) -
 -2(z^2)(x^2) e mostre que a equação A=2000 não
 possui solução
 inteira.
 A=(x^2-y^2-z^2)^2 -4y^2z^2
 A=(x^2-y^2-z^2-2yz)(x^2-y^2-z^2+2yz)
 A=(x^2-(y+z)^2)(x^2-(y-z)^2)
 A=(x+y+z)(x-y-z)(x+y-z)(x-y+z)
 Por desigualdade triangular, dah pra ver que este
 numero sempre eh negativo,
 logo A0, para x y z positivos. Acho que eh por isso
 que naum tem solucao
 pra 2000
 falow.
 marcelo

=
Rafael Werneck Cinoto
   ICQ# 107011599
 [EMAIL PROTECTED]
   [EMAIL PROTECTED]
   [EMAIL PROTECTED]
http://www.rwcinoto.hpg.com.br/

__
Do You Yahoo!?
Yahoo! Games - play chess, backgammon, pool and more
http://games.yahoo.com/
=
Instruções para entrar na lista, sair da lista e usar a lista em
http://www.mat.puc-rio.br/~nicolau/olimp/obm-l.html
O administrador desta lista é [EMAIL PROTECTED]
=


=
Instruções para entrar na lista, sair da lista e usar a lista em
http://www.mat.puc-rio.br/~nicolau/olimp/obm-l.html
O administrador desta lista é [EMAIL PROTECTED]
=



Re: [obm-l] Ajudem-me!

2002-03-06 Por tôpico Rodrigo Villard Milet

Ok desculpem-me pela euforia..
Abraços,
   Villard
-Mensagem original-
De: Paulo Jose B. G. Rodrigues [EMAIL PROTECTED]
Para: [EMAIL PROTECTED] [EMAIL PROTECTED]
Data: Terça-feira, 5 de Março de 2002 00:12
Assunto: Re: [obm-l] Ajudem-me!


 Tá doido !? Bebeu ?
 Bem.. ou vc bebeu, ou eu bebi talvez tenha sido isso...
 Q livros eu pedi !?

 Villard

Calma...
Acho que houve um pequeno engano do Professor Ângelo Barone.
Na verdade, ele respondeu mensagem enviada por [EMAIL PROTECTED]

Paulo José



 -Mensagem original-
 De: Angelo Barone Netto [EMAIL PROTECTED]
 Para: [EMAIL PROTECTED] [EMAIL PROTECTED]
 Data: Segunda-feira, 4 de Março de 2002 20:44
 Assunto: Re: [obm-l] Ajudem-me!


 Caro Rodrigo Villard Milet.
 Ha um livro de
 Chetaev:
 Theoretical Mechanics,
 traduzido para o ingles,
 que parece ter as definicoes que interessam a V.
 Eu nao conheco o livro mas me foi indicado por fonte fidedigna.
 Nao sei onde V. possa encontra-lo em SP mas na web ele esta
 anunciado em http://urss.ru por 18,4 euros.
 Espero que ajude.
 
 Angelo Barone{\ --\ }Netto   Universidade de Sao Paulo
 Departamento de Matematica Aplicada  Instituto de Matematica e
Estatistica
 Rua do Matao, 1010   Butanta - Cidade Universitaria
 Caixa Postal 66 281  phone +55-11-3091-6162/6224/6136
 05311-970 - Sao Paulo - SP   fax +55-11-3091-6131
 Agencia Cidade de Sao Paulo
 .
 
 
 
 
 
 
 
 
 
 
 

=
 Instruções para entrar na lista, sair da lista e usar a lista em
 http://www.mat.puc-rio.br/~nicolau/olimp/obm-l.html
 O administrador desta lista é [EMAIL PROTECTED]

=
 

 =
 Instruções para entrar na lista, sair da lista e usar a lista em
 http://www.mat.puc-rio.br/~nicolau/olimp/obm-l.html
 O administrador desta lista é [EMAIL PROTECTED]
 =


=
Instruções para entrar na lista, sair da lista e usar a lista em
http://www.mat.puc-rio.br/~nicolau/olimp/obm-l.html
O administrador desta lista é [EMAIL PROTECTED]
=


=
Instruções para entrar na lista, sair da lista e usar a lista em
http://www.mat.puc-rio.br/~nicolau/olimp/obm-l.html
O administrador desta lista é [EMAIL PROTECTED]
=



Re: [obm-l] Ajudem-me!

2002-03-04 Por tôpico Rodrigo Villard Milet

Tá doido !? Bebeu ?
Bem.. ou vc bebeu, ou eu bebi talvez tenha sido isso...
Q livros eu pedi !?

Villard
-Mensagem original-
De: Angelo Barone Netto [EMAIL PROTECTED]
Para: [EMAIL PROTECTED] [EMAIL PROTECTED]
Data: Segunda-feira, 4 de Março de 2002 20:44
Assunto: Re: [obm-l] Ajudem-me!


Caro Rodrigo Villard Milet.
Ha um livro de
Chetaev:
Theoretical Mechanics,
traduzido para o ingles,
que parece ter as definicoes que interessam a V.
Eu nao conheco o livro mas me foi indicado por fonte fidedigna.
Nao sei onde V. possa encontra-lo em SP mas na web ele esta
anunciado em http://urss.ru por 18,4 euros.
Espero que ajude.

Angelo Barone{\ --\ }Netto   Universidade de Sao Paulo
Departamento de Matematica Aplicada  Instituto de Matematica e Estatistica
Rua do Matao, 1010   Butanta - Cidade Universitaria
Caixa Postal 66 281  phone +55-11-3091-6162/6224/6136
05311-970 - Sao Paulo - SP   fax +55-11-3091-6131
Agencia Cidade de Sao Paulo
.











=
Instruções para entrar na lista, sair da lista e usar a lista em
http://www.mat.puc-rio.br/~nicolau/olimp/obm-l.html
O administrador desta lista é [EMAIL PROTECTED]
=


=
Instruções para entrar na lista, sair da lista e usar a lista em
http://www.mat.puc-rio.br/~nicolau/olimp/obm-l.html
O administrador desta lista é [EMAIL PROTECTED]
=



Re: [obm-l] Ajudem-me!

2002-03-01 Por tôpico Rodrigo Villard Milet

Bem, acho que só não mandaram a 2 lá vai.
Derive x^2 - xy + y^2 = 7 em relação a x. Temos que 2x - y -x*y` + 2*y*y` =
0, então segue que y` = (2x-y) / (x-2y). Como vc quer no ponto (1,3), y` =
1/5, como vc queria...

Abraços,
 Villard
-Mensagem original-
De: Felipe Pina [EMAIL PROTECTED]
Para: [EMAIL PROTECTED] [EMAIL PROTECTED]
Data: Sexta-feira, 1 de Março de 2002 07:25
Assunto: Re: [obm-l] Ajudem-me!


At 10:31 PM 2/28/2002 -0300, you wrote:
Olá pessoal!!!

Estou com alguns simples problemas que não consigo resolver e sei que
vocês
da lista conseguem.
Pretendo a transferência para a Politécnica de SP para o ano de 2003 e
para
isso estou estudando desde já. Passei, dentre outras, na UFSCar, onde
pretendo cursar e, sendo ela Federal, começará em Maio, havendo um atraso
no
ano letivo que pode me prejudicar na prova de transferência. Portanto,
conto
com a ajuda de vocês.

1) Alguém poderia me resumir sucintamente ou passar uma lista de livros
que
explicam com facilidade os conceitos de Momento de Inércia, Momento
Angular,
Torque e Rolamento?

Eu sei que estes assuntos estao bem explicados no livro do Hallyday voume 1
O nome do livro é Física I e tem uma capa preta.

2) Gostaria de saber porque a equação da reta tangente à curva
x^2-xy+y^2=7
no ponto (1,3) é 5y-x-14=0 (m=1/5) e não 6y-x-14=0 (m=1/6)   Eu
refiz a derivação mas não dá o coeficiente da resposta.

Esta n da para eu tentar agora pq estou com pressa hehe..

  /6
3)Como se Integra isto aqui :  \6x^3 * raiz(x^4+9) * dx
   /0

 essa é direta : int( 6(x^3) * raiz(x^4 + 9) * dx )
 vamos por substituicao..
 seja u = x^4 + 9, entao du = 4 * x^3 * dx
 int( 6(x^3) * raiz(x^4 + 9) * dx ) = int( raiz(u) * 6/4 * du ) =
3/2 * int( raiz(u) * du ) = 3/2 * 2/3 * u^(3/2) = u^(3/2) = (x^4 + 9)^3/2
 utilizando os limites 0 e 6 temos :
 (6^4 + 9)^(3/2) - (0^4 + 9)^(3/2) = 1305^(3/2) - 9^(3/2) =
47115.84277597183680470206363393 na minha calculadora..

4) Se for possível, mandar uma lista dos assuntos vistos em Matemática,
Física, etc, durante o primeiro ano Universitário das Engenharias no
geral.
 Que eu me lembre...

 Matematica : Algebra Linear I, Calculo I e Calculo II
 Fisica : Fisica I e Fisica II

Obrigado pela coopereção.

  Um abraço!!!


abraços,
Felipe Pina

_
Oi! Você quer um iG-mail gratuito?
Então clique aqui: http://registro.ig.com.br/censo/igmail

=
Instruções para entrar na lista, sair da lista e usar a lista em
http://www.mat.puc-rio.br/~nicolau/olimp/obm-l.html
O administrador desta lista é [EMAIL PROTECTED]
=

=
Instruções para entrar na lista, sair da lista e usar a lista em
http://www.mat.puc-rio.br/~nicolau/olimp/obm-l.html
O administrador desta lista é [EMAIL PROTECTED]
=


=
Instruções para entrar na lista, sair da lista e usar a lista em
http://www.mat.puc-rio.br/~nicolau/olimp/obm-l.html
O administrador desta lista é [EMAIL PROTECTED]
=



Re: [obm-l] Problemas afinal!!!! =)

2002-02-10 Por tôpico Rodrigo Villard Milet

1) Suponha que a(n) = r^n é solução. Então r^3 - 4r^2 + 5r - 2 = 0. Mas isso
é equivalente a r^2(r-1) -3r(r-1)+2(r-1)=0, ou seja (r-1)^2 * (r-2) = 0.
Então a gente vê que r=1 ou r = 2. É fácil notar que se algumas sequências
satisfazem a recorrência dada, então combinações lineares destas tb
satisfazem... e isso nos permite ajustar os coeficientes dessa combinação
linear, de modo que as condições iniciais sejam satisfeitas. Seja X(n) =
m*2^n + j + n*t, onde m, j e t são constantes.
 X(1) = 2m + j + t = 1
 X(2) = 4m + j + 2t = 0
 X(3) = 8m + j + 3t = -5
Daí, 2m + t = -1 e 4m + t = -5, logo m = -2 e t = 3 e j = 2.
Então X(n) = -2^(n+1) + 2 + 3n. É fácil ver que essa sequência satisfaz o
pedido...

Vou tentar mandar as outras daki a poko...
Abraços, Villard


-Mensagem original-
De: Marcelo Souza [EMAIL PROTECTED]
Para: [EMAIL PROTECTED] [EMAIL PROTECTED]
Data: Domingo, 10 de Fevereiro de 2002 16:14
Assunto: [obm-l] Problemas afinal =)


Essa lista está ficando muito monótona, sem muitas discussões sobre
problemas, só o pessoal atacando na teoria. Vou colocar alguns problemas
aqui e espero que vocês mandem soluções =)
1. Dada a sequencia infinita de inteiros a_1,a_2,..., definida por
a_1 = 1, a_2=0,a_3=-5 e a_n=4[a_(n-1)]-5[a_(n-2)]+2[a_(n-3)]   n=3
ache uma expressão fechada para a_n.
2. Prove a seguinte desigualdade:
x,y,z reais positivos, para r0
[x^r](x-y)(x-z)+[y^r](y-x)(y-z)+[z^r](z-x)(z-y)=0
Com igualdade x=y=z, ou então se dois deles forem iguais e o terceiro igual
a 0.
3.Sejam a,b,c reais positivos satisfazendo abc=1. Mostre que:
1/a^3(b+c) + 1/b^3(a+c) + 1/c^3(a+b)=3/2
valeu
abraços
Marcelo

_
MSN Photos is the easiest way to share and print your photos:
http://photos.msn.com/support/worldwide.aspx

=
Instruções para entrar na lista, sair da lista e usar a lista em
http://www.mat.puc-rio.br/~nicolau/olimp/obm-l.html
O administrador desta lista é [EMAIL PROTECTED]
=


=
Instruções para entrar na lista, sair da lista e usar a lista em
http://www.mat.puc-rio.br/~nicolau/olimp/obm-l.html
O administrador desta lista é [EMAIL PROTECTED]
=



Re: [obm-l] Problemas afinal!!!! =)

2002-02-10 Por tôpico Rodrigo Villard Milet

3) Bem, essa condição abc = 1, às vezes pede que a gente faça a=1/x, b=1/y e
c=1/z ( Lembrem do problema 2 da imo de 99 eu acho ). Ela é boa, pois ainda
temos xyz=1. Fazendo isso, queremos que :
 x^2/(y+z) + y^2/(x+z) + z^2/(x+y) = 3/2. Bem, temo quadrados do lado
maior da desigualdade... isso lembra cauchy...
Cauchy diz que (a^2+b^2+c^2)*(d^2+e^2+f^2) = (ad+be+cf)^2.
Tome : a = x/sqrt(y+z), b= y/sqrt(x+z), c=z/sqrt(x+y), d=sqrt(y+z),
e=sqrt(x+z), f=sqrt(x+y).
Então  :
[ x^2/(y+z) + y^2/(x+z) + z^2/(x+y) ] * 2(x+y+z) = (x+y+z)^2 e daí segue
que :
[ x^2/(y+z) + y^2/(x+z) + z^2/(x+y) ] = (x+y+z)/2 = 1/2 *3 raiz
cúbica(xyz)=3/2
A última desigualdade segue da desigualdade das médias...
Abraços,
 Villard
-Mensagem original-
De: Marcelo Souza [EMAIL PROTECTED]
Para: [EMAIL PROTECTED] [EMAIL PROTECTED]
Data: Domingo, 10 de Fevereiro de 2002 16:14
Assunto: [obm-l] Problemas afinal =)


Essa lista está ficando muito monótona, sem muitas discussões sobre
problemas, só o pessoal atacando na teoria. Vou colocar alguns problemas
aqui e espero que vocês mandem soluções =)
1. Dada a sequencia infinita de inteiros a_1,a_2,..., definida por
a_1 = 1, a_2=0,a_3=-5 e a_n=4[a_(n-1)]-5[a_(n-2)]+2[a_(n-3)]   n=3
ache uma expressão fechada para a_n.
2. Prove a seguinte desigualdade:
x,y,z reais positivos, para r0
[x^r](x-y)(x-z)+[y^r](y-x)(y-z)+[z^r](z-x)(z-y)=0
Com igualdade x=y=z, ou então se dois deles forem iguais e o terceiro igual
a 0.
3.Sejam a,b,c reais positivos satisfazendo abc=1. Mostre que:
1/a^3(b+c) + 1/b^3(a+c) + 1/c^3(a+b)=3/2
valeu
abraços
Marcelo

_
MSN Photos is the easiest way to share and print your photos:
http://photos.msn.com/support/worldwide.aspx

=
Instruções para entrar na lista, sair da lista e usar a lista em
http://www.mat.puc-rio.br/~nicolau/olimp/obm-l.html
O administrador desta lista é [EMAIL PROTECTED]
=


=
Instruções para entrar na lista, sair da lista e usar a lista em
http://www.mat.puc-rio.br/~nicolau/olimp/obm-l.html
O administrador desta lista é [EMAIL PROTECTED]
=



Re: [obm-l] Problemas afinal!!!! =)

2002-02-10 Por tôpico Rodrigo Villard Milet

Agora a 2, pra terminar...
2) Posso assumir que y  não é nem o maior nem o menor entre x, y e z, pois a
desigualdade é simétrica. Como x-z = (x-y)+(y-z), temos que :
[x^r](x-y)(x-z)+[y^r](y-x)(y-z)+[z^r](z-x)(z-y) =
= [x^r](x-y)^2 + [x^r](x-y)(y-z) + [y^r](y-x)(y-z) + [z^r](z-y)^2 +
[z^r](z-x)(y-x)  =
= [x^r](x-y)^2 + [z^r](z-y)^2 + (y-x)(y-z)(y^r - x^r - z^r).
Basta analisar (y-x)(y-z)(y^r - x^r - z^r).
Se x = y = z, temos que (y-x) = 0 e (y-z)=0 e y^r - x^r - z^r =0 e
segue o que queríamos..
Se z = y = x temos (y-x) = 0 e (y-z)=0 e y^r - x^r - z^r =0 e segue o
que queríamos..
Então acabou...
Abraços,
   Villard
-Mensagem original-
De: Marcelo Souza [EMAIL PROTECTED]
Para: [EMAIL PROTECTED] [EMAIL PROTECTED]
Data: Domingo, 10 de Fevereiro de 2002 16:14
Assunto: [obm-l] Problemas afinal =)


Essa lista está ficando muito monótona, sem muitas discussões sobre
problemas, só o pessoal atacando na teoria. Vou colocar alguns problemas
aqui e espero que vocês mandem soluções =)
1. Dada a sequencia infinita de inteiros a_1,a_2,..., definida por
a_1 = 1, a_2=0,a_3=-5 e a_n=4[a_(n-1)]-5[a_(n-2)]+2[a_(n-3)]   n=3
ache uma expressão fechada para a_n.
2. Prove a seguinte desigualdade:
x,y,z reais positivos, para r0
[x^r](x-y)(x-z)+[y^r](y-x)(y-z)+[z^r](z-x)(z-y)=0
Com igualdade x=y=z, ou então se dois deles forem iguais e o terceiro igual
a 0.
3.Sejam a,b,c reais positivos satisfazendo abc=1. Mostre que:
1/a^3(b+c) + 1/b^3(a+c) + 1/c^3(a+b)=3/2
valeu
abraços
Marcelo

_
MSN Photos is the easiest way to share and print your photos:
http://photos.msn.com/support/worldwide.aspx

=
Instruções para entrar na lista, sair da lista e usar a lista em
http://www.mat.puc-rio.br/~nicolau/olimp/obm-l.html
O administrador desta lista é [EMAIL PROTECTED]
=


=
Instruções para entrar na lista, sair da lista e usar a lista em
http://www.mat.puc-rio.br/~nicolau/olimp/obm-l.html
O administrador desta lista é [EMAIL PROTECTED]
=



[obm-l] Re:

2002-02-10 Por tôpico Rodrigo Villard Milet

É fácil notar ( e provar ) que a sequência muda o valor nas posições da
forma  n(n+1)/2  +  1.
Fazendo n(n+1)/2  +  1 = 1993, temos n^2 + n - 3984 =0, ou seja
0=n=62,5. Com isso percebemos que a sequencia muda de valor, pela última
vez antes de chegar no 1993º termo, no termo 62*63/2 + 1 = 1954. Então o
1993º termo é igual ao 1954º termo que é igual a 63. ( isso pq na posição
n(n+1)/2 + 1 temos o n+1 )
Logo o resto na div por 5 é 3.
Espero ñ ter errado conta..
Abraços, Villard
-Mensagem original-
De: [EMAIL PROTECTED] [EMAIL PROTECTED]
Para: [EMAIL PROTECTED] [EMAIL PROTECTED]
Data: Domingo, 10 de Fevereiro de 2002 21:23


Proponho um humilde problema :
Considere a sequencia (1,2,2,3,3,3,4,4,4,4,5,5,5,5,5,...) cujos termos
sao os inteiros consecutivos em ordem crescente e na qual o inteiro n
ocorre
n vezes. Quanto é o resto da divisao por 5 do 1993o termo desta sequencia?

Espero ter sido claro e que ele seja util para todos.

Atenciosamente,

Asselin.



--
Use o melhor sistema de busca da Internet
Radar UOL - http://www.radaruol.com.br



=
Instruções para entrar na lista, sair da lista e usar a lista em
http://www.mat.puc-rio.br/~nicolau/olimp/obm-l.html
O administrador desta lista é [EMAIL PROTECTED]
=


=
Instruções para entrar na lista, sair da lista e usar a lista em
http://www.mat.puc-rio.br/~nicolau/olimp/obm-l.html
O administrador desta lista é [EMAIL PROTECTED]
=



Re: Vestibular - ajuda

2001-12-30 Por tôpico Rodrigo Villard Milet

Na 1), basta notar que o produto das raízes é rq(6), logo a=rq(3) e como b é
a soma das raízes, b=a+rq(2), então a+b=2a+rq(2)=2*rq(3)+rq(2), B.
Na 2), veja que a função f(x)=2^x+x^2-4 é contínua. Então como f(1)=-1 e
f(2)=4, então f possui raiz entre 1 e 2. Além disso, f(-2)=1/4 e
f(-1)= -5/2, então f possui raiz entre -2 e -1.B.
-Mensagem original-
De: Thomas de Rossi [EMAIL PROTECTED]
Para: Obm-l [EMAIL PROTECTED]
Data: Domingo, 30 de Dezembro de 2001 18:22
Assunto: Vestibular - ajuda


Pessoal,

Tenho duas questões que não consigo resolver:

1) O conjunto-solução da equação em x, x^2 -bx + rq(6) = 0 é {rq(2) ; a. O
valor de a + b é
a) rq(2) + rq(3)
b) rq(2) + 2*rq(3)
c) 2*rq(2) + rq(3)
d) rq(3) + rq(6)
e) rq(2) + rq(6)

2) A equação 2^x + x^2 = 4 possui duas raízes reais a e b tais que a  b. É
correto afirmar que
a) 0  a  1 e 1  b  2
b) -2  a  -1 e 1  b  2
c) -1  a  0 e 1  b  2
d) -2  a  -1 e 0  b  1
e) -1  a  0 e 0  b  1

Estou angustiado por ter feito várias tentativas e não ter obtido êxito, se
alguém puder ajudar será um alívio.

NOTA: rq(x) : raiz quadrada de x

Desde já agradeço e um Feliz Ano Novo a todos.

Thomas.





Re: Questão

2001-12-25 Por tôpico Rodrigo Villard Milet

Para p2 vale que é inteiro sim. Eu mandei a resolução dessa pra eureka ...
vou mandar resumidamente o que eu fiz :
Primeiro cabe notar que para E = (2^(p-1)-1)/p ser quadrado, p deve
satisfazer  a afirmação : p==1mod6.
p ímpar, logo (2^(p-1)-1)==0mod3. Se p=3, então E=1 que é quadrado, logo p=3
é solução. Se p3, 3 divide E, logo 9 divide E, pois E é quadrado. Daí,
(2^(p-1)-1)==0mod9, logo 2^(p-1)==1mod9. Como 2^6==1mod9, e 6 é o menor
natural com esta propriedade ( é denominado a ordem de 2, mod9 ), então 6
divide p-1. logo está provada a afirmação.
Daí, p=1+6k entào E=(2^(6k) - 1)/p=(2^(3k)-1)(2^(3k)+1)/p.
Seja d = mdc{2^(3k)-1;2^(3k)+1}. Então d divide a diferença : d divide 2.
Como d é ímpar, d=1. Então, como 2^(3k)-1 e 2^(3k)+1 não têm fatores em
comum, um deles deve ser quadrado perfeito, enquanto o outro dividido por p
tb o deve ser.

-Caso 1: 2^(3k)-1 é quadrado.
2^(3k)-1 = (2^k-1)(2^(2k)+2^k+1). Seja h=mdc{2^k-1;2^(2k)+2^k+1}. h divide
(2^k-1)^2 e 2^(2k)+2^k+1, logo divide a diferença 3*2^(2k). Como h é ímpar,
h=1ou3. Se d=1, 2^k-1 e 2^(2k)+2^k+1 devem ser quadrados, o que é
ímpossível, pois (2^k)^2  2^(2k)+2^k+1  (2^k+1)^2, ou seja,  2^(2k)+2^k+1
está entre 2 quadrados consecutivos. Se h=3, (2^k-1)/3 é quadrado (ímpar),
logo (2^k-1)/3 ==1mod8, logo 2^k==4mod8 o que implica k=2 que não gera uma
solução.

-Caso 2: 2^(3k)+1 é quadrado.
2^(3k)+1 = q^2 ... 2^(3k) = (q+1)(q-1), logo q+1 e q-1 devem ser potências
de 2 e a única possibilidade é q=3, logo k=1 e assim, p=7, que é solução.

Logo as únicass soluções são p=3 e p=7.
Desculpem qq desatenção, verifiquem se puder.

Usei alguns fatos que podem ser desconhecidos por alguns. Naquela hora da
ordem. Se t é ordem de a modM, então se a^y==1modM, t divide y. Prova :
Divida y por t, ou seja, y=t*u+r, com rt. Daí,
a^y=a^(t*u+r)=(a^t)^u*a^r==a^r==1, mas isso só é possível se r=0, pois por
definição, t é o menor número com essa propriedade, logo t divide y. E só
mais uma coisa. x^2=1mod8, se x é ímpar. Isso decorre de
(2j+1)^2=4j(j+1) +1. como j(j+1) é par, 8 divide 4j(j+1), logo x^2==1mod8.
Abraços,
   Villard




-Mensagem original-
De: Vinicius José Fortuna [EMAIL PROTECTED]
Para: [EMAIL PROTECTED] [EMAIL PROTECTED]
Data: Terça-feira, 25 de Dezembro de 2001 14:34
Assunto: Re: Questão


Ué,

Para p=2:

(2^1 - 1)/2 = 1/2, que não é inteiro

Será que entendi errado??

Pelo exemplo entendi que a fórmula é (2^(p-1)-1)/p.
Creio que este seja um problema proposto na Eureka de setembro e a fórmula
era assim.

Qual o teorema de Euler?

Boas festas a todos!

Até mais

[ Vinicius José Fortuna  ]
[ [EMAIL PROTECTED] ]
[  Visite www.viniciusf.cjb.net  ]


On Tue, 25 Dec 2001, Henrique Lima Santana wrote:


Ae pessoal,
 deem uma olhada nessa questão
   ache todos os p, primos, tais que 2^p-1 -1/p seja um quadrado perfeito.
( 
 essa expressão resulta  sempre num n° inteiro- pelo teorema de Euler)
 -- ex: pra p=7 = 2^6 -1/7=9 q eh quadrado perf.
   valeu
 Henrique
 
 
 _
 Send and receive Hotmail on your mobile device: http://mobile.msn.com
 






Re: Semana Olímpica

2001-12-23 Por tôpico Rodrigo Villard Milet

Eu vou tb e te garanto que tem muito mais gente... o Márcio e o Arnaldo
aki da lista vão... e mais outros 3 que eu conheço !
Abraços

-Mensagem original-
De: Vinicius José Fortuna [EMAIL PROTECTED]
Para: [EMAIL PROTECTED] [EMAIL PROTECTED]
Data: Domingo, 23 de Dezembro de 2001 18:39
Assunto: Semana Olímpica


Olá Pessoal,

Gostaria de saber se vai ter muita gente do nível universitário na Semana
Olímpica. Receio chegar lá e ser o único universitário...

Até mais

[ Vinicius José Fortuna  ]
[ [EMAIL PROTECTED] ]
[  Visite www.viniciusf.cjb.net  ]







Re: Re:Re: funções e fatorial

2001-12-14 Por tôpico Rodrigo Villard Milet

Mas se não há restrições para f e g, existe uma infinidade de exemplos...
Por exemplo :
f(x) = 1, se x é par e f(x) = 0, se x é ímpar ;
g(x) = 0, se x é par e g(x) = 1, se x é ímpar .

É um exemplo muito inútil seria mais interessante você pedir f e g
contínuas
Por exemplo, tome f(n) = abs{cos[n*pi/2]} e g(n) = abs{sen[n*pi/2]}, onde
abs significa módulo. Para você perceber de onde eu tirei isso, faça o
gráfico dessas funções. Você vai ver que elas satisfazem as condições
iniciais que eu propus, antes de querer f, g contínuas em R.
Villard


-Mensagem original-
De: gabriel guedes [EMAIL PROTECTED]
Para: [EMAIL PROTECTED] [EMAIL PROTECTED]
Data: Sexta-feira, 14 de Dezembro de 2001 23:51
Assunto: Re:Re: funções e fatorial


Ola rodrigo,
gostaria de saber como vc  chegou a tal conclusão???poderia demonstrar o
racicinio???
- Original Message -
From: Rodrigo Malta Schmidt [EMAIL PROTECTED]
To: [EMAIL PROTECTED]
Sent: Friday, December 14, 2001 9:22 PM
Subject: Re: funções e fatorial



 Argh!!!

 Eu escrevi sin ao inves de cos. :)))

 sin (x*PI) = 0

 O certo entao seria:
  f(x) = cos(x * PI)
  g(x) = cos((x+1) * PI)


 Rodrigo Malta Schmidt wrote:
 
   2)ache f(x) e g(x) para:
   f(x)g(x) , se x for par
   f(x)g(x)  ,se x for impar
 
  Que tal:
 
  f(x) = sin(x * PI)
  g(x) = sin((x+1) * PI)
 
  Rodrigo






Re: ajuda

2001-12-11 Por tôpico Rodrigo Villard Milet




Sim est certo para n natural. 
No entanto podemos generalizar a demonstrao com n real 
:)
Abraos, 
 Villard

-Mensagem original-De: 
Alexandre F. Terezan [EMAIL PROTECTED]Para: 
[EMAIL PROTECTED] [EMAIL PROTECTED]Data: 
Tera-feira, 11 de Dezembro de 2001 17:56Assunto: Re: 
ajuda
No embalo do que o JP disse, de que s  
bom usar o que demonstramos, e como eu useia desigualdade 
de Bernoulli na minha solucao, a demonstracao abaixo est 
correta?

(1+x)^n = 1 + nx, para x real maior que -1, 
diferente de zero, e n natural maior que 1.

Para n = 2 -- (1+x)^2 = 1 + 2x + x^2  1 + 
2x (VERDADEIRO)

Inducao: Se vale para n, entao (1+x)^n = 1 + 
nx.

Mas (1+x)^(n+1) = (1+x)^n * (1+x)  (1+nx)(1+x) = 1 + 
(n+1)x + nx^2 1 + (n+1)x

Ou seja, se vale para n natural maior que 1, vale para 
(n+1) tambm

Como vale para n = 2, entao vale para todo n natural maior 
que 1. c.q.d.

-Mensagem Original- 
De: 
Augusto Csar Morgado 
Para: [EMAIL PROTECTED] 
Enviada em: Tera-feira, 11 
de Dezembro de 2001 11:32 Terezan
Assunto: Re: ajuda
No h dvida de que foi linda. Mas, 
supondo o sabemos que, bastaria fazer n=1. Alexandre 
F. Terezan wrote:
00c301c181e8$703c99a0$[EMAIL PROTECTED]>



Vou tentar uma sem usar 
clculo.

Desigualdade de Bernoulli: (1 + a)^n = 1 
+ an, a  -1 e n natural.

Sabemos que e^x  (1 + x/n)^n, para todo 
n

Seja a = x/n

e^x  (1 + x/n)^n -- e^x  
(1 + a)^n -- e^x  1 + an -- e^x 
 1 + x

-Mensagem Original- 

De:[EMAIL PROTECTED] 
Para: 
[EMAIL PROTECTED] 
Enviada 
em: Segunda-feira, 10 de Dezembro de 2001 00:12 
Terezan
Assunto: 
ajuda
Como se 
demonstra a desigualdade e ^ x maior ou igual a 1 + x 
?


Re: limites

2001-12-10 Por tôpico Rodrigo Villard Milet

Não entendi direito sua pergunta 1, mas parece que vc quer um jeito de
calcular o limite de sen(x)/x, qd x -0. Acho que basta usar a série para
sen(x) :
sen(x)/x = (x - x^3/3! + x^5/5! -  )/x = 1 - x^2/3 + x^4/5! - que
para x -0, vai pra 1.
Eu sei que o uso de série de potência está camuflando derivadas tb, mas não
deixa de não usar l`hôspital.

Eu concordo em parte com isso de só usar o que sabemos provar. Mas também
não podemos levar isto tanto a sério né... pois assim eu não poderia usar
relógio :)) brincadeira !
Mas vale a pena saber como demonstrar esse teoremas sim...
A regra de l`hôspital é que se f e g são funções tais que o limite f(x)/g(x)
( com x tendendo a a ) é indeterminado do tipo 0/0, então este limite é
igual ao limite de f`(x)/g`(x), com x -a .
Bem, com as hipóteses, temos que f(a) = g(a) = 0. Logo, lim[f(x)/g(x)] =
lim[f(x) - f(a)/g(x)-g(a)] = lim[(f(x) - f(a))/(x-a)] / lim
[(g(x)-g(a))/(x-a)] = lim f`(x)/g`(x).

É claro que f e g devem ser deriváveis e também é claro que podemos dividir
por x-a no limite, pois o limite é tomado numa vizinhaça furada de a, logo
x-a é diferente de zero.
JP, você está certo nisso sim... tenho quase certeza de que mais de 90% das
pessoas que cursam cálculo 1 não tentaram demonstrar ou viram a demonstração
da regra acima... isso não deveria ser assim... mas...


Abraços,
 Villard

-Mensagem original-
De: Jose Paulo Carneiro [EMAIL PROTECTED]
Para: [EMAIL PROTECTED] [EMAIL PROTECTED]
Data: Segunda-feira, 10 de Dezembro de 2001 23:33
Assunto: Re: limites


cotg ^(1/log)  eh o inverso de tg^(1/log) = e^(ln tg x / ln x).
Quando x-0 (pela direita, eh claro), ln tg x e ln x tendem ambos
a -infinito.
vale L'Hopital: o quociente das derivadas eh
(sec^2 x / tg x) / (1/x) = x / sen x cos x - 1.
Logo o limite eh: 1/e
(se nao houver erro de conta)

Quanto ao segundo, uma variante, para variar:
a derivada de e^x para x=0 eh sabido = 1.
Esta derivada, por definicao, eh e^h - 1 / h quando h- 0.
Substituindo h por 2x (por que vale?):
e^(2x)-1 / 2x tende a 1.
Logo e^(2x)-1 / x tende a 2.

[Sempre que posso, evito usar L'Hopital, por 2 motivos:
1) muitas vezes, o uso de l'Hopital esconde o uso da propria definicao de
derivada. exemplo:
sen x / x  quando x tende a 0. Por l'Hopital, cos x / 1 tende a 1. mas como
voce sabe que a derivada de sen x eh cos x, se nao souber que senx / x
tende
a 1? alguem conhece um jeito?
2) Alguem ahi ja demonstrou l'Hopital? Eu so gosto de usar aquilo que algum
dia demonstrei.
Ih, ja sei que vai dar polemica...]

JP



- Original Message -
From: Hugo Iver Vasconcelos Goncalves [EMAIL PROTECTED]
To: [EMAIL PROTECTED]
Sent: Monday, December 10, 2001 8:57 PM
Subject: Re: limites


confere com o que eu tinha achado sim... valeu vinicius e juliana
e quanto à primeira vcs encontraram algo?

- Original Message -
From: Vinicius José Fortuna [EMAIL PROTECTED]
To: [EMAIL PROTECTED]
Sent: Monday, December 10, 2001 6:12 PM
Subject: Re: limites


On Mon, 10 Dec 2001, Hugo Iver Vasconcelos Goncalves wrote:

 qual o limite das seguintes funções?

 lim (cotgx)^(1/lnx)
 x- 0


 lim (e^2x -1)/x
 x-0

Essa eu acho que sei:

lim{x-0} (e^2x - 1)/x =
lim{x-0} (e^2x)/x - 1/x =
lim{x-0} (e^2x)/x
Por L'Hopital (é assim que se escreve?)
= lim{x-0} 2.(e^2x) + 2x.(e^2x) =
= 2

Confere?

Até mais

Vinciius










Re: ajuda

2001-12-09 Por tôpico Rodrigo Villard Milet




Use um pouquinho de 
Clculo ...
Considere f(x) = e^x - (1+x). Da, f `(x) = e^x - 1. f ` (x) = 0 
implica x=0.  fcil notar que x=0  minimante de f, pois f 
``(0) = 1 0.
Ento f(0) = 0  o menor valor que f(x) assume, logo f(x) = 
e^x - (1+x) =0, e segue-se que e^x = 1+x :))

Abraos, 
 Villard
-Mensagem original-De: [EMAIL PROTECTED] [EMAIL PROTECTED]Para: [EMAIL PROTECTED] [EMAIL PROTECTED]Data: 
Segunda-feira, 10 de Dezembro de 2001 00:36Assunto: 
ajuda
Como se demonstra a desigualdade e ^ 
x maior ou igual a 1 + x ? 


Re: Isolando X

2001-12-07 Por tôpico Rodrigo Villard Milet

Faz o seguinte : se f é estritamente crescente e g é estritamente
decrescente, então f=g adimite no máximo uma solução. O lado esquerdo é
decrescente e o direito é crescente, logo só há uma solução, que você acha
por inspeção, x=1/2.

-Mensagem original-
De: romenro [EMAIL PROTECTED]
Para: [EMAIL PROTECTED] [EMAIL PROTECTED]
Data: Sexta-feira, 7 de Dezembro de 2001 15:09
Assunto: Isolando X


Estou com uma tremenda dificuldade em isolar o X dessa
questão:


(1/4)^x=x,

a resposta é 1/2, mas fiz por substituição.

Se alguém puder me dar uma ajuda desde jé agradeço.

Rodrigo



__
Quer ter seu próprio endereço na Internet?
Garanta já o seu e ainda ganhe cinco e-mails personalizados.
DomíniosBOL - http://dominios.bol.com.br






Re: pascal

2001-12-01 Por tôpico Rodrigo Villard Milet

Claro !  C(n,p) = n!/[p!*(n-p)!]   :)
Villard
-Mensagem original-
De: pichurin [EMAIL PROTECTED]
Para: [EMAIL PROTECTED] [EMAIL PROTECTED]
Data: Domingo, 2 de Dezembro de 2001 00:07
Assunto: pascal


gostaria de saver se existe alguma fórmula para
determinar qualqier termo do triângulo de pascal sem
ter que montá-lo.

___

Yahoo! GeoCities
Tenha seu lugar na Web. Construa hoje mesmo sua home page no Yahoo!
GeoCities. É fácil e grátis!
http://br.geocities.yahoo.com/





Re: Funções... (Iezzi)

2001-11-26 Por tôpico Rodrigo Villard Milet




f(x+1)=2f(x) + 3, logo f(x+1) + t = 2*[f(x) + t/2 +3/2]. Ento basta 
fazer t = t/2 +3/2, ou seja, t=3, pois fazendo s(x) = f(x) + 3, temos s(x+1) 
=2*s(x), logo s(x)=2^x * s(0). Como s(0)=f(0)+3=3, temos f(x)=s(x) - 3 = 3*2^x - 
3 = 3*[2^x - 1]...
Villard

-Mensagem original-De: 
Luis Lopes [EMAIL PROTECTED]Para: 
[EMAIL PROTECTED] [EMAIL PROTECTED]Data: 
Segunda-feira, 26 de Novembro de 2001 18:28Assunto: Re: 
Funes... (Iezzi)
Sauda,c~oes,

 
f(n + 1) = 2f(n) + 3

Suponha f(n+1) = 2f(n). 
Ento temos uma PG de razo 2.

f(n) = k_1 2^n. Levando em considerao o 
termo constante,
corrigimos nosso termo geral:

f(n) = k_1 2^n + k_2.

Para obter k_2, fazemos f(n) = k_2. Ento f(n+1) = 
k_2 e k_2 = -3.

Com f(0) = 0, obtemos 0 = k_1 - 3. Logo k_1 = 
3.

Assim, f(n) = 3(2^n - 1). Verifique por 
induo.

[]'s
Lus


-Mensagem Original- 
De: 
Davidson Estanislau 
Para: obm 

Enviada em: Segunda-feira, 26 de 
Novembro de 2001 17:10
Assunto: Funes... 
(Iezzi)


 Caros amigos, encontrei a seguinte 
funo:

 f(n) = 6(2^(n - 1) 
- 0.5)

 Por induo temos:

 Para n=0:

 f(0) = 6(2^(0-1) - 0.5) = 
0

 Supondo que a expresso, f(n) = 
6(2^(n-1)-0.5), seja verdadeira. 

 Para n+1, teremos:

 f(n+1) = 2f(n) + 3 = 2*6(2^(n 
- 1) - 0.5) + 3 = 6(2^n - 1) + 3 = 6[(2^n - 1) + 0.5] = 6[2^2 - 
0.5] c.q.d.

 
 Abs!!!
 

 Davidson Estanislau
-Mensagem 
Original-De: {O-Grande-Mentecapto} [EMAIL PROTECTED]Para: 
[EMAIL PROTECTED]Enviada 
em: Domingo, 25 de Novembro de 2001 16:11 TerezanAssunto: 
Funes... 
(Iezzi) 
Ol.. 
Estou aqui resolvendo um problema de funes do Iezzi, mas 
comopara esse tipo de exerccio 'dissertativo' no 
h resposta nas ltimaspginas, no sei 
se cheguei a soluo correta.Seja f uma 
funo, definida no conjunto dos nmeros naturais, 
tal 
que: 
f(n + 1) = 2f(n) + 3para todo n natural.a) Supondo f(0) = 0, 
calcule f(1),f(2),f(3),f(4),... e descubra a 
frmulageral de f(n).b) Prove por 
induo finita a frmula descoberta.(in 
IEZZI, Gelson FME vol 1. pp 157)Fazendo f(1), f(2), f(3) etc.. 
achamos:f(1) = 3, f(2) = 9, f(3) = 21,f(4) = 45, f(5) = 93 ... f(n) 
= ?expandindo as contas, temos:f(1) = (0.2) + 
3f(2) = (((0.2) + 3).2) + 3f(3) = 0.2) + 3).2) + 3).2) + 
3f(4) = (0.2) + 3).2) + 3).2) + 3).2)+3f(5) = ((0.2) + 
3).2) + 3).2) + 3).2)+3).2 + 3Tomando n = 3 e 
desenvolvendo:f(3) = 3.2.2 + 3.2 + 3o mesmo para n = 4:f(4) 
= 3.2.2.2 + 3.2.2 + 3.2 + 3ou 3.2 + 3.2 + 3.2 + 
3Isso decorre de que n+1  dado por n.2 + 3..Colocando o 
3 em evidncia.. e notando que a maior potncia de 2 
 igual an-1:f(n) = 3(2^(n-1) + 2^(n-2) +  + 
2^1 + 2^0)ou ainda f(n) = 3. somatria[para k = 0 at 
n - 1] 2^kA frmula funciona para qualquer n pertencente aos 
naturais e diferente dezero.Da que vem minha 
dvida... a frmula que eu achei pode ser 
considerada'termo' geral, se no  vlida para 
0? Algum tem 
alguma idia de outra frmula geral?Grato 
pela ateno..Against stupidity, 
the Gods themselves contend in vain, 
Friedrich von Schiller's-[]'s{O-Grande-Mentecapto}[EMAIL PROTECTED] 



Re: sistema

2001-11-02 Por tôpico Rodrigo Villard Milet




Olhe para o sistema como se as variveis fossem 
apenas x e y e tente elimin-las.
Vamos chamar a primeira equao de 
(I) e a segunda de (II).
Faa (I)cosq - (II)senq : y*[(cosq)^2 + (senq)^2] = 2a*sen2q*cosq - 
a*cos2q*senq, logo y = 4a*senq*[1-(senq)^2] - a*[1-2(senq)^2]*senq = 3a*senq - 
2a*(senq)^3.

Faa (I)senq + (II)cosq : x*[(senq)^2 + (cosq)^2] = 2a*sen2q*senq + 
a*cos2q*cosq, logo x = 4a*cosq*[1-(cosq)^2] + a*[2(cosq)^2-1]*cosq = 3a*cosq - 
2a*(cosq)^3.

Hum... Note que as expresses de x e y so bem parecidas... 
vamos ento calcular x+y e x-y .
Mas antes, note que (senq)^3 +- (cosq)^3 = (senq +- cosq)[1-+ 
(sen2q)/2]

x+y = 3a*(senq + cosq) - 2a*[(senq)^3 + (cosq)^3] = 
a*(senq+cosq)[3-2[1-(sen2q)/2] = a*(senq+cosq)[3-2[1-(sen2q)/2] = a*(senq + 
cosq)*(1+sen2q)
Hum... mas (senq + cosq)^2 = 1+sen2q... 
ento deve ser legal calcular (x+y)^2 :
(x+y)^2 = a^2 * (1+sen2q) * (1+sen2q)^2 
= a^2 * (1+sen2q)^3
Analogamente, (x-y)^2 = a^2 * 
(1-sen2q)^3 :))
Ento temos : 1+sen2q = (x+y)^(2/3) / a^2
 
e 1-sen2q = (x-y)^(2/3) / a^2
Somando : (x+y)^(2/3) 
/ a^2 + (x-y)^(2/3) / a^2 = 2, logo :

 (x+y)^(2/3) + (x-y)^(2/3) = 
2a^2.

Abraos, Villard !

-Mensagem original-De: 
Eduardo Azevedo [EMAIL PROTECTED]Para: 
[EMAIL PROTECTED] [EMAIL PROTECTED]Data: 
Sexta-feira, 2 de Novembro de 2001 17:50Assunto: 
sistema

Calcule a em funo de x e y no 
sistema:

x.senq+y.cosq=2a.sen2q
x.cosq-y.senq=a.cos2q


Re: sistema

2001-11-02 Por tôpico Rodrigo Villard Milet




desculpe o erro na ltima msg... 
 a^(2/3)..
Villard

-Mensagem original-De: 
Eduardo Azevedo [EMAIL PROTECTED]Para: 
[EMAIL PROTECTED] [EMAIL PROTECTED]Data: 
Sexta-feira, 2 de Novembro de 2001 17:50Assunto: 
sistema

Calcule a em funo de x e y no 
sistema:

x.senq+y.cosq=2a.sen2q
x.cosq-y.senq=a.cos2q


Função

2001-10-28 Por tôpico Rodrigo Villard Milet




Determine todas as funes f:R-R, 
tais que f(f(f(x)))=f(f(x))+f(x)+x, para todo x real.
Villard


Re: Relação de Euler ( poliedros )

2001-10-25 Por tôpico Rodrigo Villard Milet

A demonstração mais comum e natural disso é fazendo por indução no número de
faces do poliedro... não é difícil não, mas tem que ter paciência. Se você
quiser eu digito, mas tem que falar...
 Abraços,
Villard
-Mensagem original-
De: Gustavo Nunes Martins [EMAIL PROTECTED]
Para: [EMAIL PROTECTED] [EMAIL PROTECTED]
Data: Quinta-feira, 25 de Outubro de 2001 20:00
Assunto: Re: Relação de Euler ( poliedros )


Eu posso te mostrar, mas so garanto que da certo quando as faces so estao
em planos perpendiculares ao fundo e ao topo do poliedro (estes devem
estar parelelos entre si). Depois te mando um e-mail.

Adiantando, acho que fica mais facil se vc fizer V+F = A+2 e pensar como
vc constroi as Arestas a partir dos Vertices e, depois, as Faces a partir
das arestas. Comeca pensando numa caixa em que a base e o topo sao
triangulos iguals.

René Retz wrote:

 Alguem sabe provar a relaçao:

 Em todo poliedro convexo, ou em toda superfície poliedrica fechada, é
 valida a relação: V - A + F = 2

 onde:
 V = nº de vértices
 A = nº de arestas
 F = nº de vértices

 Desculpe o incomodo.
René





Re: OBM-u

2001-10-22 Por tôpico Rodrigo Villard Milet

   E aí, Márcio ! Pô, como eu já tinha falado contigo antes, qd cheguei em
casa fiz de um jeito bem parecido com o seu, na força bruta mesmo. Mas na
hora da prova eu fiz usando 2 funções, pra ver se montava uma recorrência e
montei :) O problema é que eu errei em um pedacinho, aí os erros de conta
foram carregados até o final... é uma pena...
   Ah, eu queria saber se alguém poderia dar uma idéia pra 6. Eu cheguei a
tentar um pouco na prova, e tentei mostrar algumsa coisas. Primeiro eu vi
que o bordo ficava fixo. Mas não consegui provar que o centro era fixo, o
que dificultou muito... Quando eu olhei pra essa questão, achei que tinha a
ver com o teorema dos pontos fixos das contrações... é, aquilo era uma
contraçào somente quando valia a desigualdade estrita. Daí, eu supus por
Ultra contradição que valia a desigualdade estrita para todos e a partir
daí tentei ver para quais pontos isso era imopssível ( queria concluir que
não era possível para nenhum, né ). Para o bordo é óbvio... daí, pelo
teorema dos pontos fixos das contrações, existe um único ponto no disco D,
tal que f(a)=a, ou seja, apenas um ponto ficaria parado. Mas aí não consegui
formalizar minha idéia a partir daí.
 Abraços
   Villard
-Mensagem original-
De: Marcio [EMAIL PROTECTED]
Para: [EMAIL PROTECTED] [EMAIL PROTECTED]
Data: Terça-feira, 23 de Outubro de 2001 00:57
Assunto: Re: OBM-u


Oi Bruno! Td bom? Tb achei a prova legal.. Qto ao resultado, acho que
fiz a 1 e a 5, nao completei direito a 2 pq nao lembrava exatamente do
enunciado (ou prova) de um teorema que tinha na Eureka 3 (no artigo de
fracoes continuas) que me ajudaria muito. Na 4, que eu achei uma questao
bem
interessante, eu tmb
escrevi.
Fiquei um tempao, umas 2h30m ou mais escrevendo nela, mas ja descobri q
errei uma bobagem na solucao.. Na hora achei logo uma recorrencia que
parecia facilitar a coisa e acreditei nela.. Ela se mostrou util, mas eh
provavelmente uma maneira bem horrivel de se fazer a questao.. Acabou
demorando bem mais do que eu imaginava ...
E voce, como foi na prova? Quais voce conseguiu fazer? Como foi o
pessoal ai na USP? Do pessoal que eu conversei que fez a prova comigo,
parece que a maioria foi mais ou menos igual a mim, acertando umas duas (O
pessoal acertava em geral a 1 e a 4).
Alem da 3, ainda nao consegui enxergar nada muito interessante na 6.
 Depois me mostraram algumas solucoes bem mais legais pra dois (a
melhor
que eu vi ateh agora criava 2 sequencias auxiliares para trelicas
semelhantes a do problema e ai ficava bem simples.. )
Bom, depois de reescrever tudo aqui no papel, ver onde eu tinha me
enganado,
e achar a nova resposta, nao aguentei e digitei aqui pra mandar pra lista
tmb! Espero que vc e mais alguem alguem tenha paciencia de ler e/ou
comentar! :)
Minhas ideias na dois seguem no proximo email!

Gostaria ainda de deixar uma pergunta sobre a questao 5. Eu consegui
fazer a letra (b) usando a (a), mas hoje o
Luciano me disse que tinha uma solucao legal do Nicolau integrando no plano
complexo, que eu acabei esquecendo de perguntar.. Alguem (ou o proprio
Nicolau) pode me mostrar como? Na prova, antes da (a), eu cheguei a tentar
olhar pra integral como uma integral complexa no semicirculo de raio
1(substituindo cosx = z + 1/z), mas nao sabia como achar os polos daquela
funcao para poder integrar..

Marcio

- Original Message -
From: Bruno Fernandes Cerqueira Leite [EMAIL PROTECTED]
To: [EMAIL PROTECTED]
Sent: Monday, October 22, 2001 10:52 PM
Subject: Re: OBM
 ...
 Acho que a Nelly também cometeu erros tipográficos na questão 3 do nível
 universitário! :-)

 Essa questão era beem difícil, eu até agora não sei como fazer um
 avanço não trivial.

 Aliás, ninguém parabenizou ainda a prova da universitária, então eu vou
 parabenizar: estava muito boa, as questões eram muito bonitas: valeu a
pena
 pensar 9 horas nelas! (eu até pensaria mais) Como vcs foram?

 Bruno Leite

 []s, N.
 
 






Re: Probleminha de Geometria Analítica

2001-10-21 Por tôpico Rodrigo Villard Milet

Olhe sua msg abaixo...
-Mensagem original-
De: Fernando Henrique Ferraz [EMAIL PROTECTED]
Para: [EMAIL PROTECTED] [EMAIL PROTECTED]
Data: Domingo, 21 de Outubro de 2001 03:08
Assunto: Probleminha de Geometria Analítica


Estou cá me debatendo com um problema aparentemente simples de GA, vejamos:

(Cesgranrio 1990) Determine o comprimento do segmento cujos extremos são os
pontos de intersecção do círculo x² + y² = 2 com a parábola y = x².

Bom, basta achar os valores de x e y que satisfaçam ao mesmo tempo as duas
equações. Se y = x², joguei ele na primeira equação:
y + y² = 2
y² + y - 2 = 0
resolvendo por bhaskara: y'= 1 e y'' = -2

Pois bem.. y' = 1... x² = y, x = 1

AQUI, NA VERDADE É X=+-1, DAÍ AS DUAS SOLUÇÕES...
PARA Y=-2, NÃO HÁ SOLUÇÕES ! :)


O problema vem no segundo resultado: y'' = -2.. x² = -2... x = sqrt(2)i!!
Mas se x é imaginário, a parábola não deveria cortar a circunferência em
dois pontos, apenas em um, tangenciando.
Para piorar as coisas, fiz o gráfico da situação no Equantion Graph, e ele
confirmou os dois encontros (em anexo...).
Onde está o erro, nos meus cálculos ou no exercício e no graph eq. ?






Against stupidity, the Gods themselves contend in vain,
 Friedrich von Schiller's
-
[]'s
Fernando Henrique Ferraz / {O-Grande-Mentecapto]
[EMAIL PROTECTED]






Re: Desigualdades (correcao)

2001-10-17 Por tôpico Rodrigo Villard Milet

grau zero, pois fica t^0. a da IMO era assim :

a/sqrt(a^2+8bc) + b/sqrt(b^2+8ac) + c/sqrt(c^2+8ba) =1

Quando vc calcula F(ta,tb,tc) dá exatamente F(a,b,c), ou seja o grau é zero.
Você pode olhar no livro Problem-Solving Strategies, Arthur Engel. Ele dá
mais um exemplo :
Prove que a,b,c0  a/(b+c) + b/(a+c) + c/(a+b) = 3/2.
Ele mesmo diz que F(a,b,c) é homogênea de grau 0 e nesse caso ele faz a
normalização a+b+c=1, que é bem natural pois aparecem as somas parciais.
Então use a desigualdade das médias aritmética e harmônica com os números
a+b, a+c, b+c. Daí (a+b+a+c+b+c)(1/(a+b) + 1/(a+c) + 1/(b+c)) =9... ou seja
:  1/(a+b) + 1/(a+c) + 1/(b+c) = 9/2 logo (a+b+c)/(a+b) + (a+b+c)/(a+c) +
(a+b+c)/(b+c) =9/2 ...
 a/(b+c) + b/(a+c) + c/(a+b) = 3/2 (CQD)
 Talvez quando vc perceber que a desigualdade é homogênea vc nem tenha que
citar o grau... a substituição que faz é o mais importante...
 Mas vc percebeu pq pode normalizar ? é que (a,b,c) satisfaz a
desigualdade se, e somente se, (ta,tb,tc) tb satisfaz (isso é bastante claro
qd o grau é zero). Daí, se vc supõe por exemplo a+b+c = 1 e prova a
desigualdade, vc está provando q vale a desigualdade para qualquer soma
a+b+c. Mesma coisa para o artifício a=1, b=1+x, c=1+y... basta olhar para o
que acontece com o a.
Me corrijam se estiver errado.
Abraços, Villard

PS.: Em qual livro vc está estudando isso ?



-Mensagem original-
De: Marcelo Souza [EMAIL PROTECTED]
Para: [EMAIL PROTECTED] [EMAIL PROTECTED]
Data: Quarta-feira, 17 de Outubro de 2001 14:09
Assunto: Re: Desigualdades (correcao)


cara, outra coisa que nao tinha reparadoto mandando agora...
Eu acho que naum existe homogenea de grau zerode acordo com o livro
f(a,b,c)= f(ta,tb,tc)=tf(a,b,c), com t dif de zero, o minimo e grau 1.
valeu!
corrijam se eu estiver errado
abracos
M.



From: Rodrigo Villard Milet [EMAIL PROTECTED]
Reply-To: [EMAIL PROTECTED]
To: [EMAIL PROTECTED]
Subject: Re: Desigualdades
Date: Wed, 17 Oct 2001 00:03:10 -0300

Uma desigualdade é dita simétrica se ao trocar de ordem as variáveis a
desigualdade não se altera.
Ex.: a^2 + b^2 + c^2 = ab+ac+bc.
OBS: É interessante termos uma desigualdade simétrica nas variáveis, pois
podemos supor sem perda de generalidade que elas estão numa certa ordem.
No
exemplo que eu dei, vc pode supor a =b =c ( é claro que há 1001 maneiras
de provar essa desigualdade sem isso ).

Agora, vamos olhar para desigualdades de outra maneira. Deixe todas as
variáveis de um lado da inequação. Desse lado tem-se uma função de várias
variáveis.
Ex.: Em a^2 + b^2 + c^2 = ab+ac+bc, faça F(a,b,c) = a^2 + b^2 + c^2 -
ab-ac-bc. Vc quer provar que F(a,b,c)=0, para quaisquer a,b,c.
Uma função é dita homogênea de grau n, quando f(ta,tb,tc)=t^n * f(a,b,c).
A desigualdade acima é então homogênea  de grau 2.

Eu acho que o grau não importa muito. O que interessa é se ela é homogênea
ou não.
Por exemplo, na desigualdade acima, note que F(ta,tb,tc)=0 se e somente
se
F(a,b,c)=0. Então podemos fazer algumas normalizações ( fizar a soma das
variáveis, fixar uma das variáveis, etc...).
No exemplo dado, faça a=1, b=1+x, c=1+y. Ficamos com
F(1,1+x,1+y)=x^2+y^2-xy=(x-y/2)^2 + (3y^2)/4 =0.

Outro exemplo bastante significativo é o problema 2 desta última IMO. Era
uma desigualdade homogênea ( de grau 0, o que não importa ). Daí, era
legal
fazer a+b+c=1, o que nos possibilitava usar a desigualdade de Jensen... e
assim vai. A moral da história é : fique feliz se a desigualdade for
simétrica ou homogênea, pois você ou pode matar o problema direto, ou pode
cair num problema mais fácil. :)

Espero não ter errado alguma definição,
Abraços,
Villard
 -Mensagem original-
 De: Marcelo Souza [EMAIL PROTECTED]
 Para: [EMAIL PROTECTED] [EMAIL PROTECTED]
 Data: Terça-feira, 16 de Outubro de 2001 19:40
 Assunto: Desigualdades


 ol[a pessoal,

 Quando que uma desigualdade e simetrica (acho que diz simetrica em
relacao as variaveis)?

 Quando uma desigualdade e homogenea de grau n?

 abracos



--
--
 Get your FREE download of MSN Explorer at http://explorer.msn.com



_
Get your FREE download of MSN Explorer at http://explorer.msn.com/intl.asp





Re: Desigualdades

2001-10-16 Por tôpico Rodrigo Villard Milet




Uma desigualdade  dita simtrica se ao 
trocar de ordem as variveis a desigualdade no se 
altera.
Ex.: a^2 + b^2 + c^2 = ab+ac+bc.
OBS:  interessante termos uma desigualdade simtrica nas 
variveis, pois podemos supor sem perda de generalidade que elas 
esto numa certa ordem. No exemplo que eu dei, vc pode supor a =b 
=c (  claro que h 1001 maneiras de provar essa desigualdade 
sem isso ).

Agora, vamos olhar para desigualdades de outra maneira. 
Deixe todas as variveis de um lado da inequao. Desse 
lado tem-se uma funo de vrias variveis. 

Ex.: Em a^2 + b^2 + c^2 = ab+ac+bc, faa 
F(a,b,c) = a^2 + b^2 + c^2 - ab-ac-bc. Vc quer provar que F(a,b,c)=0, para 
quaisquer a,b,c.
Uma funo  dita 
homognea de grau n, quando f(ta,tb,tc)=t^n * f(a,b,c). 
A desigualdade acima  ento homognea de grau 
2.

Eu acho que o grau no importa muito. O que interessa  se 
ela  homognea ou no.
Por exemplo, na desigualdade acima, note que F(ta,tb,tc)=0 se e somente 
se F(a,b,c)=0. Ento podemos fazer algumas 
normalizaes ( fizar a soma das variveis, fixar uma das 
variveis, etc...).
No exemplo dado, faa a=1, b=1+x, c=1+y. Ficamos com 
F(1,1+x,1+y)=x^2+y^2-xy=(x-y/2)^2 + (3y^2)/4 =0.

Outro exemplo bastante significativo  o problema 2 desta 
ltima IMO. Era uma desigualdade homognea ( de grau 0, o que 
no importa ). Da, era legal fazer a+b+c=1, o que nos 
possibilitava usar a desigualdade de Jensen... e assim vai. A moral da 
histria  : fique feliz se a desigualdade for simtrica ou 
homognea, pois voc ou pode matar o problema direto, ou pode cair 
num problema mais fcil. :)

Espero no ter errado alguma definio, 
Abraos, 
Villard

-Mensagem original-De: 
Marcelo Souza [EMAIL PROTECTED]Para: 
[EMAIL PROTECTED] [EMAIL PROTECTED]Data: 
Tera-feira, 16 de Outubro de 2001 19:40Assunto: 
Desigualdades

ol[a pessoal,
Quando que uma desigualdade e 
simetrica (acho que diz simetrica em relacao as variaveis)?
Quando uma desigualdade e 
homogenea de grau n?
abracos

Get your FREE download of MSN Explorer at http://explorer.msn.com


Re: Teorema de galois

2001-10-14 Por tôpico Rodrigo Villard Milet

Pelo q eu saiba não há prova simples pra esse teorema... é todo um assunto
da álgebra.
 Villard
-Mensagem original-
De: Carlos Maçaranduba [EMAIL PROTECTED]
Para: [EMAIL PROTECTED] [EMAIL PROTECTED]
Data: Domingo, 14 de Outubro de 2001 18:45
Assunto: Teorema de galois


alguém poderia dar uma prova simples como funciona o
teorema de galois relativo a representação das raizes
de um polinomio em função de seus coeficientes.Pq a
partir do 5 grau não existe formula assim como existe
a fómula de baskára para o 2 grau???

___

Yahoo! GeoCities
Tenha seu lugar na Web. Construa hoje mesmo sua home page no Yahoo!
GeoCities. É fácil e grátis!
http://br.geocities.yahoo.com/





Re: Funcao exponencial

2001-10-06 Por tôpico Rodrigo Villard Milet

Se vc considerar A0, o que fazer com (-2)^(1/2) ??? e se A=1, a função
permanece constante... sakô ?
 Villard
-Mensagem original-
De: [EMAIL PROTECTED] [EMAIL PROTECTED]
Para: [EMAIL PROTECTED] [EMAIL PROTECTED]
Data: Sábado, 6 de Outubro de 2001 21:58
Assunto: Re: Funcao exponencial


Na minha opinião o A maior que zero e diferente de um é uma questão de
ajuste
da função exponencial a fenomënos fisicos que cresçam e decreçam
exponencialmenteuma base negativa teria mesmas imagens para valores
pares
e simétricos de x, o que não caracterizariam uma função exponencial( ja
ouviu
a frase...* a economia tem melhorado exponencialmente*?)...É uma questão de
ajuste creio eu
Vou esperar outras explicações, pois como disse, o que falei acima reflete
minha opinião pessoal
Um Abraço...
   Ruy





Notas de corte !?

2001-09-21 Por tôpico Rodrigo Villard Milet




Sei que j perguntei uma vez, mas estou meio ansioso pra saber quel 
ser a nota de corte para a segunda fase da OBM Universitria 
se no souberem, tm alguma previso ???
Obrigado, 
 Villard


Universitária

2001-09-06 Por tôpico Rodrigo Villard Milet




E a, pessoal, ningum vai falar sobre a 
OBM universitria ?? Digam como vocs foram, por favor... bem, eu 
fiz as questes 1, 3 e 4 inteiras e na 6 eu tirei algumas 
concluses... Ah, e se algum tiver algum palpite pra qual deve 
ser a nota de corte, tb  bem aceito.
Abraos, 
 
Villard


Re: 2 QUESTÕES

2001-08-30 Por tôpico Rodrigo Villard Milet

Este B que eu disse é B={n E N*;f(x)=x+k, k E N}
-Mensagem original-
De: Henrique Lima [EMAIL PROTECTED]
Para: [EMAIL PROTECTED] [EMAIL PROTECTED]
Data: Terça-feira, 28 de Agosto de 2001 23:04
Assunto: 2 QUESTÕES


OLÁ,
Gostaria da ajuda de vcs nas seguintes questões:
1.Os numeros positivos x,y e z são tais que:
x=2y/1+y , y=2z/1+z e z=2x/1+x.
prove q x=y=z
2. Determine todas as funções estritamente crescentes f:N*-N* tais que
f(n+f(n)=2f(n)
  valeu!



_
Get your FREE download of MSN Explorer at http://explorer.msn.com/intl.asp





Re: 2 QUESTÕES

2001-08-30 Por tôpico Rodrigo Villard Milet

E onde escrevi logo vemos que, segue n0 E B-2n0+k E B.
O E é pertence.
-Mensagem original-
De: Henrique Lima [EMAIL PROTECTED]
Para: [EMAIL PROTECTED] [EMAIL PROTECTED]
Data: Terça-feira, 28 de Agosto de 2001 23:04
Assunto: 2 QUESTÕES


OLÁ,
Gostaria da ajuda de vcs nas seguintes questões:
1.Os numeros positivos x,y e z são tais que:
x=2y/1+y , y=2z/1+z e z=2x/1+x.
prove q x=y=z
2. Determine todas as funções estritamente crescentes f:N*-N* tais que
f(n+f(n)=2f(n)
  valeu!



_
Get your FREE download of MSN Explorer at http://explorer.msn.com/intl.asp





Re: 2 QUESTÕES

2001-08-30 Por tôpico Rodrigo Villard Milet

Esta solução pode parecer bastante densa, mas tente acompanhar passo a passo
com papel e lápis ao lado. Há uma maneira mais informal de fazê-la, mas
enxerguei este jeito e fui até o final :)).
(I) f(n + f(n)) = 2f(n)
- Como f é crescente e f : N*-N* , nota-se que f(n)=n
Seja n0 o menor elemento de B. Logo, por (I), temos :
f(n0 + n0 + k) = 2n0 + 2k - f(2n0 + k) = (2n0+k) + k
Logo, vemos que :
- Vamos provar por indução que 2^(g)n0 + (2^(g)-1)k pertence a B :
Para g = 1 está provado. Supondo provado para g e substituindo em (I), temos
: f(2^(g)n0 + (2^(g)-1)k + 2^(g)n0 + 2^(g)k) = 2^(g+1)n0 + 2^(g+1)k -
f(2^(g+1)n0 + (2^(g+1)-1)k = 2^(g+1)n0 + 2^(g+1)k, o que prova o pedido.
- Analisemos um intervalo entre 2 elementos de B :
2^(g)n0 + (2^(g)-1)k  y  2g+1n0 + (2^(g+1)-1)k
Como a  b - f(a)  f(b), temos 2^(g)n0 + 2^(g)k  f(y)  2^(g+1)n0 +
2^(g+1)k.
Nesse intervalo, temos o mesmo número de possibilidades para y e f(y), logo
afirmamos que f é bijetora. Se f(y)  y + k, para algum y, teríamos, para t
 y, mais possibilidades para t do que para f(t), o que contradiria f ser
estritamente crescente. Analogamente, provamos que não podemos ter para
nenhum y, f(y)  y + k. Daí, verificamos que f(y) = y + k satisfaz o pedido.
Resposta : f(n) = n + k, com k natural.

Villard


-Mensagem original-
De: Henrique Lima [EMAIL PROTECTED]
Para: [EMAIL PROTECTED] [EMAIL PROTECTED]
Data: Terça-feira, 28 de Agosto de 2001 23:04
Assunto: 2 QUESTÕES


OLÁ,
Gostaria da ajuda de vcs nas seguintes questões:
1.Os numeros positivos x,y e z são tais que:
x=2y/1+y , y=2z/1+z e z=2x/1+x.
prove q x=y=z
2. Determine todas as funções estritamente crescentes f:N*-N* tais que
f(n+f(n)=2f(n)
  valeu!



_
Get your FREE download of MSN Explorer at http://explorer.msn.com/intl.asp





Re: Equação

2001-08-22 Por tôpico Rodrigo Villard Milet




Eu acho que sua questo era : Achar TODAS 
as solues inteiras pra essa equao... n 
??
Villard

-Mensagem original-De: 
Davidson Estanislau [EMAIL PROTECTED]Para: 
obm [EMAIL PROTECTED]Data: 
Segunda-feira, 20 de Agosto de 2001 14:19Assunto: 
Equao
 Achar x, y e z inteiros que 
satisfaam a seguinte equao:

 

Davidson


Re: questão de geometria

2001-08-19 Por tôpico Rodrigo Villard Milet




Os permetros dos tringulos ABC e ABD so iguais, logo AC+BC=AD+BD.
Os perimetros dos tringulos ACD e BCD so iguais, logo 
AC+AD=BC+BD.
Somando essas equaes, temos AC=BD.
Subtraindo essas equaes, temos BC=AD.
Da, os tringulos ACD e BCD so congruentes 
ang(CAD)=ang(CBD)... o quadriltero #ABCD  inscritvel. 
Da, ang(ABD)=ang(BAC), pois esto inscritos em arcos congruentes. 
Com isso, o tringulo ABO  issceles... AO=BO. Da mesma 
forma, CO=DO.

Abraos, Villard !


-Mensagem original-De: 
Odelir Maria Casanova dos Santos [EMAIL PROTECTED]Para: 
[EMAIL PROTECTED] [EMAIL PROTECTED]Data: 
Domingo, 19 de Agosto de 2001 18:49Assunto: questo de 
geometria
 Tudo bem pessoal, a 
vai uma questo :

 As diagonais AC e BD de um 
quadriltero ABCD cortam-se num ponto O. Os permetros dos 
tringulos ABC e ABD so iguais, como tambm so 
iguais os perimetros dos tringulos ACD e BCD. Mostre que AO = 
BO


Re: Re: Vamos contar?

2001-08-17 Por tôpico Rodrigo Villard Milet

É verdade... foi só o detalhe de eu ter escrito que era um real... é que eu
escrevi pensando em outra coisa, mas a idéia é a mesma...
 Villard
-Mensagem original-
De: Eduardo Casagrande Stabel [EMAIL PROTECTED]
Para: [EMAIL PROTECTED] [EMAIL PROTECTED]
Data: Quinta-feira, 16 de Agosto de 2001 03:05
Assunto: Re: Re: Vamos contar?


Rodrigo, essa demonstraccao nao esta certa. O fato de A nao pertencer a
Im(U), nao implica que a funccao U nao e' sobrejetiva, por que o conjunto X
pode nem possuir um numero real! Voce adaptou a sua demonstraccao anterior,
mas esqueceu que agora voce esta lidando com o conjunto X, e nao com os
reais. Eu acho que voce quis dizer o seguinte:

Suponha que existe uma bijeccao U: X-P(X). Construimos o conjunto A = {x
pertence X| x nao pertence a U(x)}.

Como a funccao U e' uma bijeccao, existe um x pertencente a X, tal que U(x)
= A. Agora usamos a definiccao de A:
- se x pertence a A, entao x nao pertence a U(x) = A, logo nao pertence a
A,
absurdo!
- se x nao pertence a A, entao x pertence a U(x) = A, logo pertence a A,
novo absurdo!

Ou seja, a suposiccao inicial de que existe uma bijeccao é falsa, e daí
#P(X)  #X.

Mas a sua idéia é simples e elegante! Eu nunca havia visto essa
demonstraccao. Veja como eu demonstrei que existem infinitos tipos de
infinitos.

Seja F(X) = { f: X-X }, ou seja, F(X) é o conjunto de TODAS as funções de
X
em X. Vou mostrar que #F(X)  #X.

É trivial que #F(X) = X. Temos que mostrar que nao vale a igualdade. Vou
mostrar que nao existe uma função sobrejetora U: X-F(X) (uma função que
assuma todos os valores de F(X)).

Suponha que existe essa sobrejeção U: X-P(X). Sejam a e b dois elementos
distintos de X. Para cada x pertencente a X, U(x) é uma função f_x : X-X.
Consideremos a função u: X-X, definida por:
- u(x) = a, se f_x(x) = b
- u(x) = b, se f_x(x) é diferente de b
A propriedade da função u é que ela assume um valor diferente de f_x, no
ponto x. Logo a função u é diferente de todas as funções f_x, para qualquer
x pertencente a X. Segue que U não é uma sobrejeção, um absurdo que
demonstra que #F(X)  #X.

Eduardo Casagrande Stabel.



From: Rodrigo Villard Milet [EMAIL PROTECTED]
 É verdade. Dado um conjunto X, mostramos que #(P(X))  #(X). Vejamos :
  É trivial que #(P(X)) = #(X) ( inclusão natural ). Basta mostrar que
não

 vale a igualdade. Bem, como na minha outra mensagem, suponha que exista
um
 bijeção U : X-P(X). Daí, considere o conjunto A = { y real ; y não
pertence
 a U(y) } ( obviamente A não é vazio ). Afirmação : A não pertence a
Im(U).
 Suponha o contrário. Daí, existe t real, tal que U(t) = A.
  Se t pertence a A, pela definição de A, t não pertence a U(t) = A,
 contradição.
  Se t não pertence a A, t não pertence a U(t), logo, pela definição de A,
t
 pertence a A, contradição. Daí, conclui-se que este t não existe. Logo, A
 não pertence a Im(U). Com isso, temos que a função U não é sobrejetiva,
 logo, não há bijeção de X em P(X), daí #(P(X))  #(X).
  Abraços,
   ¡Villard!



 -Mensagem original-
 De: Nicolau C. Saldanha [EMAIL PROTECTED]
 Para: [EMAIL PROTECTED] [EMAIL PROTECTED]
 Data: Quarta-feira, 15 de Agosto de 2001 08:35
 Assunto: Re: Vamos contar?


 On Tue, Aug 14, 2001 at 05:19:15PM -0300, Bruno Mintz wrote:
  Olá...
 
  Fiquei sabendo ontem de uma coisa muito divertida... :) Não sei se
  coisa é uma palavra tão ruim assim, porque infinitos são mesmo
  coisas(!) não muito bem definidas.  É o seguinte: para contar, por
  exemplo, quantas bananas existem num cacho, eu associo um número
natural
  a uma das bananas e exatamente àquela banana o mesmo número, certo?
  (Correspondência biunívoca.) Pergunta: quantos números naturais
existem?
  Infinitos... a gente sempre pode pôr mais um. Oquei... Quantos
inteiros?
  Bacana a resposta: tantos quantos os naturais... Basta associar a cada
  inteiro um natural. E racionais? Idem.  E reais? Aí não é o mesmo: tem
  mais...
 
 Isto que você descreve são cardinais infinitos, conforme estudados
 inicialmente por Cantor. Dois conjuntos X e Y tem o mesmo cardinal se
 existe uma bijeção entre eles; o cardinal de X é menor ou igual ao de Y
 se existir uma função injetora de X para Y, ou, equivalentemente,
 se existir uma função sobrejetora de Y para X.
 
  Para a quantidade de naturais, o professor usou a letra (hebraica)
  aleph com o índice zero: A_0. Para a quantidade de reais, usou A_1 e
  disse que é possível demostrar que existem infinitos tipos de
infinitos!
  Um maior que o outro! A_2, por exemplo, ele associou a alguma
propriedade
  do espaço funcional.
 
 Sinto muito contradizer seu professor, mas esta não é a notação usual.
 Aleph_0 é sempre o cardinal dos naturais mas Aleph_1 é por definição
 o cardinal seguinte. A hipótese do contínuo diz que o cardinal de R
 é Aleph_1; a hipótese do contínuo é independente dos axiomas usuais
 da teoria dos conjuntos e os especialistas discordam quanto a se ela
 deve ser considerada intuitivamente verdadeira ou falsa. Se uma

Re: Vamos contar?

2001-08-15 Por tôpico Rodrigo Villard Milet

Consulte o livro do Halmos ou o livro do Elon de Análise. Posso mostrar que
os Reais não são enumeráveis, ou seja, que não podemos contar os reais.
Para isso, temos que mostrar que não existe bijeção de N em R, ok ? Bem,
suponha que esta bijeção existe. Daí, vou mostrar que eskecemos sempre de
contar pelo menos um real, e daí a função não é sobrejetiva. Para isso,
considere que ao representar um número real, ao invés de colocarmos os
algarismos de 0 a 9, colocamos suas representações binárias. Agora, cada
real é representado por seqüências de 0`s e 1`s ( Obviamente essa
representação não tem unicidade, por exemplo 110 pode ser 6, 30 ou 12..
Mas o que interessa é que cada real possui UMA representação desta forma
! ).  Para evitar problemas futuros, vamos estabeleces que dado um certo
número de reais, vou querer que todos tenham a mesma quantidade de dígitos à
esquerda da vírgula, para isso, complete com zeros à esquerda quando
necessário. Vamos então explicitar a bijeção f: N-R :
 f(1) = a(1_1)a(1_2)a(1_3)_...,b(1_1)b(1_2)
 f(2) = a(2_1)a(2_2)a(2_3)_...,b(2_1)b(2_2)

 f(n) = a(n_1)a(n_2)a(n_3)_...,b(n_1)b(n_2)
.

Seja X o número real, tal que X = c(1_1)c(2_2)c(3_3)_... onde :
 c(i_i) = 0, se a(i_i) = 1
 c(i_i) = 1, se a(i_i) = 0
Note que X não pertence à Im(f), se tivéssemos X = f(y), para algum y, então
certamente pela construção de X, temos c(y_y) diferente de a(y_y), logo X é
diferente de f(y), para qualquer y, logo, X não pertence à Im(f).
Com isso, deixamos de contar esse X e assim f não é sobrejetiva !
 Logo, #(R)  #(N)

Abraços,  ¡ Villard !
-Mensagem original-
De: Caio Augusto [EMAIL PROTECTED]
Para: [EMAIL PROTECTED] [EMAIL PROTECTED]
Data: Terça-feira, 14 de Agosto de 2001 22:41
Assunto: Re: Vamos contar?


Ola Bruno,

Nunca ajudei na lista pq num da tempo, sempre alguem resopnde antes q eu,
mas acho q isso pode ajudar a entender o infinito dos irracionais e talvez
ateh dos complexos. Pelo que meu professor me disso eh obvio q da pra
associar os naturais aos inteiros. Mas a partir dai complica.
Para associar os naturais aos racionais, basta lembrar que o racional eh da
forma p/q, ligar a dupla (p,q) de inteiros aos irracionais. Jah para os
irracionais eh impossivel, pq se pudessemos escrever uma enumeracao de
todos
irracionais, Cantor mostrou que conseguiria mostrar um irracional que naum
estivesse na lista. E assim o infinito dos irracionais eh muito maior q o
dos racionais, e portanto a probabilidade de sortear um racional entre 0 e
1, eh 0 e de sortear um irracional eh 1. Talvez entaum para associar os
inteiros aos complexos precisamos de uma quadrupla (p,q,x,y) ligando ao
complexo (p/q)+(x/y)i.
Qualquer coisa q eu deixei meio vago me avisem pq naum sou mto bom em
explicacoes.

Caio Augusto





Re: Vamos contar?

2001-08-15 Por tôpico Rodrigo Villard Milet

É verdade. Dado um conjunto X, mostramos que #(P(X))  #(X). Vejamos :
 É trivial que #(P(X)) = #(X) ( inclusão natural ). Basta mostrar que não
vale a igualdade. Bem, como na minha outra mensagem, suponha que exista um
bijeção U : X-P(X). Daí, considere o conjunto A = { y real ; y não pertence
a U(y) } ( obviamente A não é vazio ). Afirmação : A não pertence a Im(U).
Suponha o contrário. Daí, existe t real, tal que U(t) = A.
 Se t pertence a A, pela definição de A, t não pertence a U(t) = A,
contradição.
 Se t não pertence a A, t não pertence a U(t), logo, pela definição de A, t
pertence a A, contradição. Daí, conclui-se que este t não existe. Logo, A
não pertence a Im(U). Com isso, temos que a função U não é sobrejetiva,
logo, não há bijeção de X em P(X), daí #(P(X))  #(X).
 Abraços,
  ¡Villard!



-Mensagem original-
De: Nicolau C. Saldanha [EMAIL PROTECTED]
Para: [EMAIL PROTECTED] [EMAIL PROTECTED]
Data: Quarta-feira, 15 de Agosto de 2001 08:35
Assunto: Re: Vamos contar?


On Tue, Aug 14, 2001 at 05:19:15PM -0300, Bruno Mintz wrote:
 Olá...

 Fiquei sabendo ontem de uma coisa muito divertida... :) Não sei se
 coisa é uma palavra tão ruim assim, porque infinitos são mesmo
 coisas(!) não muito bem definidas.  É o seguinte: para contar, por
 exemplo, quantas bananas existem num cacho, eu associo um número natural
 a uma das bananas e exatamente àquela banana o mesmo número, certo?
 (Correspondência biunívoca.) Pergunta: quantos números naturais existem?
 Infinitos... a gente sempre pode pôr mais um. Oquei... Quantos inteiros?
 Bacana a resposta: tantos quantos os naturais... Basta associar a cada
 inteiro um natural. E racionais? Idem.  E reais? Aí não é o mesmo: tem
 mais...

Isto que você descreve são cardinais infinitos, conforme estudados
inicialmente por Cantor. Dois conjuntos X e Y tem o mesmo cardinal se
existe uma bijeção entre eles; o cardinal de X é menor ou igual ao de Y
se existir uma função injetora de X para Y, ou, equivalentemente,
se existir uma função sobrejetora de Y para X.

 Para a quantidade de naturais, o professor usou a letra (hebraica)
 aleph com o índice zero: A_0. Para a quantidade de reais, usou A_1 e
 disse que é possível demostrar que existem infinitos tipos de infinitos!
 Um maior que o outro! A_2, por exemplo, ele associou a alguma propriedade
 do espaço funcional.

Sinto muito contradizer seu professor, mas esta não é a notação usual.
Aleph_0 é sempre o cardinal dos naturais mas Aleph_1 é por definição
o cardinal seguinte. A hipótese do contínuo diz que o cardinal de R
é Aleph_1; a hipótese do contínuo é independente dos axiomas usuais
da teoria dos conjuntos e os especialistas discordam quanto a se ela
deve ser considerada intuitivamente verdadeira ou falsa. Se uma
demonstração
usa a hipótese do contínuo, esta hipótese deve ser claramente enunciada
e surge naturalmente a questão se existe uma demonstração que não use
a hipótese do contínuo.

Algumas notações usualmente aceitas para o cardinal dos reais são
2^{Aleph_0} e Beth_0 (Beth é a segunda letra do alfabeto hebraico).

 Pergunta: Quantos complexos há? Tantos quanto os reais? Mais?

Existem exatamente tantos complexos quanto reais.
Uma idéia ingênua é escrever parte real e parte imaginária
como expansões decimais infinitas e intercalar os dígitos
para obter um único número real que 'encodifica' os dois primeiros.
O problema é que como 1.... = 0....
(conforme já foi bastante discutido nesta lista ;-))
às vezes um número real admite duas expansões decimais.
O problema é contornável de várias formas.

 Como demostrar
 que existem infinitos tipos de infinito?

Você pode mostrar que nunca existe uma bijeção entre um conjunto X e
P(X) = {Y | Y é subconjunto de X}. Mais, o cardinal de P(X) é sempre
maior do que o de X.

 (Talvez fosse interessante alguém
 reproduzir a demonstração do que eu disse acima, porque eu não saberia
 explicá-la bem. Se não me engano, o matemático que estudou isto foi
 Canton(?), Cantor(?),...)

 Não sei se essas idéias podem sair da matemática pura (podem???),

De onde mais?

 mas todos temos, no mínimo, curiosidade quando falamos do infinito.

Claro.

[]s, N.





Re: Vamos contar?

2001-08-15 Por tôpico Rodrigo Villard Milet

Quanto à questão dos complexos, você deve decidir que complexos você quer,
se Z[i], Q[i], R[i]... Os dois primeiros são enumeráveis... o terceiro não.
 Abraços,
 !Villard!
-Mensagem original-
De: Caio Augusto [EMAIL PROTECTED]
Para: [EMAIL PROTECTED] [EMAIL PROTECTED]
Data: Terça-feira, 14 de Agosto de 2001 22:41
Assunto: Re: Vamos contar?


Ola Bruno,

Nunca ajudei na lista pq num da tempo, sempre alguem resopnde antes q eu,
mas acho q isso pode ajudar a entender o infinito dos irracionais e talvez
ateh dos complexos. Pelo que meu professor me disso eh obvio q da pra
associar os naturais aos inteiros. Mas a partir dai complica.
Para associar os naturais aos racionais, basta lembrar que o racional eh da
forma p/q, ligar a dupla (p,q) de inteiros aos irracionais. Jah para os
irracionais eh impossivel, pq se pudessemos escrever uma enumeracao de
todos
irracionais, Cantor mostrou que conseguiria mostrar um irracional que naum
estivesse na lista. E assim o infinito dos irracionais eh muito maior q o
dos racionais, e portanto a probabilidade de sortear um racional entre 0 e
1, eh 0 e de sortear um irracional eh 1. Talvez entaum para associar os
inteiros aos complexos precisamos de uma quadrupla (p,q,x,y) ligando ao
complexo (p/q)+(x/y)i.
Qualquer coisa q eu deixei meio vago me avisem pq naum sou mto bom em
explicacoes.

Caio Augusto





Re: Mais notas

2001-07-10 Por tôpico Rodrigo Villard Milet

Professor, as pontuações finais saem hj mesmo ??
 Villard!
-Mensagem original-
De: Nicolau C. Saldanha [EMAIL PROTECTED]
Para: [EMAIL PROTECTED] [EMAIL PROTECTED];
[EMAIL PROTECTED] [EMAIL PROTECTED]
Data: Terça-feira, 10 de Julho de 2001 15:52
Assunto: Mais notas




Resultados parciais (oficiais):

  p1  p2  p3  p4  p5  p6   tot
Bra1   -   -   0   -   2   0
Bra2   -   -   0   -   7   0
Bra3   -   -   0   -   7   0
Bra4   -   -   0   -   2   0
Bra5   -   -   0   -   7   0
Bra6   -   -   0   -   2   0

Bem, as notas nos problemas 3 e 6 n~ao foram especialmente boas,
mas os problemas que ainda faltam devem ter notas melhores...
[]s, N.





  1   2   >